You are on page 1of 56

MATHEMATICS

Target IIT JEE

PROBABILITY
Theory and Exercise Booklet

CONTENT

S.No. Topic Page No.

1. Theory 01-25

2. Level - 1 26-33

3. Level - 2 34-37

4. Level - 3 38-39

5. Exercise - 1 40-42

6. Exercise - 2 42-45

7. Exercise - 3 45-53

8. Answers 53-55
PROBABILITY
There are various phenomena in nature, leading to an outcome, which cannot be predicted apriori
e.g. in tossing of a coin, a head or a tail may result. Probability theory aims at measuring the
uncertainties of such outcomes.
(I) Important terminology:
(i) Random Experiment :

It is a process which results in an outcome which is one of the various possible outcomes that are
known to us before hand e.g. throwing of a die is a random experiment as it leads to fall of one
of the outcome from {1, 2, 3, 4, 5, 6}. Similarly taking a card from a pack of 52 cards is also a random
experiment.

(ii) Sample Space :

It is the set of all possible outcomes of a random experiment e.g. {H, T} is the sample space associated
with tossing of a coin.
In set notation it can be interpreted as the universal set.

Solved Example # 1
Write the sample space of the experiment ‘A coin is tossed and a die is thrown’.
Solution
The sample space S = {H1, H2, H3, H4, H5, H6, T1, T2, T3, T4, T5, T6}.
Solved Example # 2
Write the sample space of the experiment ‘A coin is tossed, if it shows head a coin tossed again
else a die is thrown.
Solution
The sample space S = {HH, HT, T1, T2, T3, T4, T5, T6}
Solved Example # 3
Find the sample space associated with the experiment of rolling a pair of dice (plural of die) once. Also find
the number of elements of the sample space.
Sol. Let one die be blue and the other be grey. Suppose ‘1’ appears on blue die and ‘2’ appears on grey die. We
denote this outcome by an ordered pair (1, 2). Similarly, if ‘3’ appears on blue die and ‘5’ appears on grey die,
we denote this outcome by (3, 5) and so on. Thus, each outcome can be denoted by an ordered pair (x, y),
where x is the number appeared on the first die (blue die) and y appeared on the second die (grey die). Thus,
the sample space is given by
S = {(x, y) x is the number on blue die and y is the number on grey die}
We now list all the possible outcomes (figure)

1 2 3 4 5 6
1 (1, 1) (1, 2) (1, 3) (1, 4) (1, 5) (1, 6)
2 (2, 1) (2, 2) (2, 3) (2, 4) (2, 5) (2, 6)
3 (3, 1) (3, 2) (3, 3) (3, 4) (3, 5) (3, 6)
4 (4, 1) (4, 2) (4, 3) (4, 4) (4, 5) (4, 6)
5 (5, 1) (5, 2) (5, 3) (5, 4) (5, 5) (5, 6)
6 (6, 1) (6, 2) (6, 3) (6, 4) (6, 5) (6, 6)

Figure
Probability [1]
Number of elements (outcomes) of the above sample space is 6 × 6 i.e., 36

Self Practice Problems :


1. A coin is tossed twice, if the second throw results in head, a die is thrown.
Answer {HT, TT, HH1, HH2, HH3, HH4, HH5, HH6, TH1, TH2, TH3, TH4, TH5, TH6}.
2. An urn contains 3 red balls and 2 blue balls. Write sample space of the experiment ‘Selection of a
ball from the urn at random’.
Answer {R1, R2, R3, B1, B2 }.
Note : -
Here the balls are distinguished from one and other by naming red balls as R1, R2 and R3 and the blue
balls as B1 and B2.
(iii) Event :
It is subset of sample space. e.g. getting a head in tossing a coin or getting a prime number is
throwing a die. In general if a sample space consists ‘n’ elements, then a maximum of 2 n events
can be associated with it.

(iv) Complement of event :


The complement of an event ‘A’ with respect to a sample space S is the set of all elements of ‘S’ which
are not in A. It is usually denoted by A, A or AC.

(v) Simple Event :


If an event covers only one point of sample space, then it is called a simple event e.g. getting a head
followed by a tail in throwing of a coin 2 times is a simple event.

(vi) Compound Event :


When two or more than two events occur simultaneously, the event is said to be a compound event.
Symbolically A  B or AB represent the occurrence of both A & B simultaneously.

Note : “A B” or A + B represent the occurrence of either A or B.

Solved Example # 4
Write down all the events of the experiment ‘tossing of a coin’.
Solution
S = {H, T}
the events are  , {H}, {T}, {H, T}

Solved Example # 5
A die is thrown. Let A be the event ‘ an odd number turns up’ and B be the event ‘a number divisible
by 3 turns up’. Write the events (a) A or B (b) A and B
Solution
A = {1, 3, 5}, B = {3, 6}
 A or B = A  B = {1, 3, 5, 6}
A and B = A  B = {3}

Probability [2]
Self Practice Problems :

3. A coin is tossed and a die is thrown. Let A be the event ‘H turns up on the coin and odd number turns
up on the die’ and B be the event ‘ T turns up on the coin and an even number turns up on the die’.
Write the events (a) A or B (b) A and B.
Answer (a) {H1, H3, H5, T2, T4, T6} (b) 

4. In tossing of two coins, let A = {HH, HT} and B = {HT, TT}. Then write the events (a) A or B
(b) A and B.
Answer (a) {HH, HT, TT} (b) {HT}

(vii) Equally likely Events :


If events have same chance of occurrence, then they are said to be equally likely.
e.g
(i) In a single toss of a fair coin, the events {H} and {T} are equally likely.
(ii) In a single throw of an unbiased die the events {1}, {2}, {3} and {4}, are equally likely.
(iii) In tossing a biased coin the events {H} and {T} are not equally likely.

(viii) Mutually Exclusive / Disjoint / Incompatible Events :

Two events are said to be mutually exclusive if occurrence of one of them rejects the possibility of
occurrence of the other i.e. both cannot occur simultaneously.
In the vein diagram the events A and B are mutually exclusiv e. Mathematically, we write
AB=

Solved Example # 6
In a single toss of a coin find whether the events {H}, {T} are mutually exclusive or not.
Solution
Since {H}  {T} = ,
 the events are mutually exclusive.

Solved Example # 
In a single throw of a die, find whether the events {1, 2}, {2, 3} are mutually exclusive or not.
Solution
Since {1, 2}  {2, 3} = {2} 
 the events are not mutually exclusive.

Self Practice Problems :

5. In throwing of a die write whether the events ‘Coming up of an odd number’ and ‘Coming up of an even
number’ are mutually exclusive or not.
Answer Yes

6. An experiment involves rolling a pair of dice and recording the numbers that come up. Describe the following
events :
A : the sum is greater than 8.

Probability [3]
B : 2 occurs on either die.
C : the sum is at least 7 and a multiple of 3.
Also, find A  B, B  C and A  C.
Are (i) A and B mutually exclusive ?
(ii) B and C mutually exclusive ?
(iii) A and C mutually exclusive ?
Ans. A = {(3, 6), (4, 5), (5, 4), (6, 3), (4, 6), (5, 5), (6, 4), (5, 6), (6, 5), (6, 6)}
B = {(1, 2), (2, 2), (3, 2), (4, 2), (5, 2), (6, 2), (2, 1), (2, 3), (2, 4). (2, 5), (2, 6)}
C = {(3, 6), (6, 3), (5, 4), (4, 5), (6, 6)}
A  B = , B  C = , A  C = {(3, 6), (6, 3), (5, 4), (4, 5), (6, 6)}
(i) Yes (ii) Yes (iii) No.

(ix) Exhaustive System of Events :


If each outcome of an experiment is associated with at least one of the events E 1, E2, E3, .........E n ,
then collectively the events are said to be exhaustive. Mathematically we write
E1 E2 E3.........En = S. (Sample space)

Solved Example # 8
In throwing of a die, let A be the event ‘even number turns up’, B be the event ‘an odd prime turns
up’ and C be the event ‘a numbers less than 4 turns up’. Find whether the events A, B and C form
an exhaustive system or not.
Solution
A  {2, 4, 6}, B  {3, 5} and C  {1, 2, 3}.
Clearly A B C = {1, 2, 3, 4, 5, 6} = S. Hence the system of events is exhaustive.

Solved Example # 9
Three coins are tossed. Describe
(i) two events A and B which are mutually exclusive
(ii) three events A, B and C which are mutually exclusive and exhaustive.
(iii) two events A and B which are not mutually exclusive.
(iv) two events A and B which are mutually exclusive but not exhaustive.
(v) three events A, B and C which are mutually exclusive but not exhaustive.
Ans. (i) A : “getting at least two heads” B : “getting at least two tails”
(ii) A : “getting at most one heads” B : “getting exactly two heads”
C : “getting exactly three heads”
(iii) A : “getting at most two tails” B : “getting exactly two heads”
(iv) A : “getting exactly one head” B : “getting exactly two heads”
(v) A : “ getting exactly one tail” B : “getting exactly two tails”
C : “getting exactly three tails”
[Note : There may be other cases also]

Self Practice Problems :

7. In throwing of a die which of the following pair of events are mutually exclusive ?
(a) the events ‘coming up of an odd number’ and ‘coming up of an even number’
(b) the events ‘coming up of an odd number’ and ‘coming up of a number  4’
Answer (a)

8. In throwing of a die which of the following system of events are exhaustive ?


(a) the events ‘an odd number turns up’, ‘a number  4 turns up’ and ‘the number 5 turns up’.
(b) the events ‘a number  4 turns up’, ‘a number > 4 turns up’.
(c) the events ‘an even number turns up’, ‘a number divisible by 3 turns up’, ‘number 1 or 2
turns up’ and ‘the number 6 turns up’.

Probability [4]
Answer (b)

( II) Classical (A priori) Definition of Probability :

If an experiment results in a total of (m + n) outcomes which are equally likely and mutually exclusive
wi t h one a not her and i f ‘m ’ ou t com es are f av orabl e t o an ev ent ‘A’ whi l e ‘n’ are
unf av orabl e, t hen t he probab i l i t y of occurrence of t he ev ent ‘A’, denot ed by

m number of favourable outcomes


P(A), is defined by = total number of outcomes
mn

m
i.e. P(A) = .
mn
We say that odds in favour of ‘A’ are m : n, while odds against ‘A’ are n : m.

n
Note that P( A ) or P(A) or P(AC), i.e. probability of non-occurrence of A = = 1 – P(A)
mn
In the above we shall denote the number of out comes favourable to the event A by n(A) and the total
number of out comes in the sample space S by n(S).

n( A )
 P(A) = .
n(S)
Solved Example # 10
In throwing of a fair die find the probability of the event ‘ a number  4 turns up’.
Solution
Sample space S = {1, 2, 3, 4, 5, 6} ; event A = {1, 2, 3, 4}
 n(A) = 4 and n(S) = 6

n( A ) 4 2
 P(A) = = = .
n(S) 6 3
Solved Example # 11
In throwing of a fair die, find the probability of turning up of an odd number  4.
Solution
S = {1, 2, 3, 4, 5, 6}
Let E be the event ‘turning up of an odd number  4’
then E = {5}

n (E) 1
 P(E) = n (S) = .
6
Solved Example # 12
In throwing a pair of fair dice, find the probability of getting a total of 8.
Solution.
When a pair of dice is thrown the sample space consists
{(1, 1) (1, 2) .......... (1, 6)
(2, 1,) (2, 2,)......... (2, 6)
.... ..... .... ...
.... ... ... ...
(6, 1), (6, 2) ........ (6, 6)}
Note that (1, 2) and (2, 1) are considered as separate points to make each outcome as equally likely.
To get a total of ‘8’, favourable outcomes are, (2, 6) (3, 5) (4, 4) (5, 3) and (6, 2).

Probability [5]
5
Hence required probability =
36

Solved Example # 13
A four digit number is formed using the digits 0, 1, 2, 3, 4 without repetition. Find the probability that it is
divisible by 4
Solution

Total 4 digit numbers formed

Each of these 96 numbers are equally likely & mutually exclusive of each other.
Now, A number is divisible by 4, if last two digits of the number is divisible by 4
Hence we can have  first two places can be filled in 3 × 2 = 6 ways

 first two places can be filled in 2 × 2 = 4 ways

 6 ways

 4 ways

 4 ways

 6 ways
__________
Total number of ways 30 ways
favorable outcomes 30 5
probability = Total outcomes
= = Ans.
96 16

Self Practice Problems :

9. A bag contains 4 white, 3 red and 2 blue balls. A ball is drawn at random. Find the probability of the
event (a) the ball drawn is white or red (b) the ball drawn is white as well as red.
Answer (a) 7/9 (b) 0

10. In throwing a pair of fair dice find the probability of the events ‘ a total of of less than or equal to 9”’.
Answer 5/36.

(III) Addition theorem of probability :

If ‘A’ and ‘B’ are any two events associated with an experiment, then

P(AB) = P(A) + P(B) – P(AB)

De Morgan’s Laws : If A & B are two subsets of a universal set U, then


(a) (A B)c = Ac Bc
(b) (A B)c = Ac Bc
Distributive Laws : (a) A  (B C) = (A B)  (A C)
(b) A  (B C) = (A B)  (A C)

For any three events A, B and C we have the figure

Probability [6]
(i) P(A or B or C) = P(A) + P(B) + P(C) – P(A B) – P(B C) – P(C A) + P(A B C)
(ii) P (at least two of A, B, C occur) = P(B  C) + P(C  A) + P(A  B) – 2P(A B C)
(iii) P(exactly two of A, B, C occur) = P(B C) + P(C A) + P(A B) – 3P(A B  C)
(iv) P(exactly one of A, B, C occur) =
P(A) + P(B) + P(C) – 2P(B C) – 2P(C A) – 2P(A B) + 3P(A B C)

Note : If three events A, B and C are pair wise mutually exclusive then they must be mutually exclusive,
i.e. P(A  B) = P(B C) = P(C A) = 0  P(A B C) = 0.
However the converse of this is not true.

Solved Example # 14
A bag contains 4 white, 3red and 4 green balls. A ball is drawn at random. Find the probability of the
event ‘the ball drawn is white or green’.
Solution
Let A be the event ‘the ball drawn is white’ and B be the event ‘the ball drawn is green’.

8
P(The ball drawn is white or green) = P (A  B) = P(A) + P(B) – P(A  B) =
11

Solved Example # 15
In throwing of a die, let A be the event ‘an odd number turns up’, B be the event ‘a number divisible
by 3 turns up’ and C be the event ‘a number  4 turns up’. Then find the probability that exactly two
of A, B and C occur.
Solution
Event A = {1, 3, 5}, event B = {3, 6} and event C = {1, 2, 3, 4}
 A  B = {3}, B  C = {3}, A  C = {1, 3} and A  B  C = {3}.
Thus P(exactly two of A, B and C occur)
= P(A  B) + P(B  C) + P(C  A) – 3P(A  B  C)

1 1 2 1 1
= + + –3× =
6 6 6 6 6

Self Practice Problems :


11. In throwing of a die, let A be the event ‘an odd number turns up’, B be the event ‘a number divisible
by 3 turns up’ and C be the event ‘a number  4 turns up’. Then find the probability that atleast two
of A, B and C occur.

1
Answer
3

Probability [7]
12. In the problem number 11, find the probability that exactly one of A, B and C occurs.

2
Answer
3

(IV) Conditional Probability

P(A  B)
If A and B are two events, then P(A/B) = .
P(B)
Note that for mutually exclusive events P(A/B) = 0.

Solved Example # 16
If P(A/B) = 0.2 and P(B) = 0.5 and P(A) = 0.2. Find P(A  B ).
Solution.
P(A  B ) = P(A) – P(A  B)

P( A  B)
Also P(A/B) =
P(B)
 P(A  B) = 0.1
From given data,
P(A  B ) = 0.1

Solved Example # 17
If P(A) = 0.25, P(B) = 0.5 and P(A  B) = 0.14, find probability that neither ‘A’ nor ‘B’ occurs. Also find

P AB 
Solution
 
We have to find P A  B = 1 – P(A  B) (by De-Morgan’s law)
Also, P(A B) = P(A) + P(B) – P(A  B)

putting data we get, P A  B  = 0.39

The shaded region denotes the simultaneous occurrence of A and B

 
Hence P A  B = P(A) – P(A  B) = 0.11

Self Practice Problem:-

13. If P( A / B ) = 0.2, P(A  B) = 0.9, then find P(A  B ) ?


Ans. 0.4

5. Independent and dependent events


If two events are such that occurence or non-occurence of one does not affect the chances of occurence
or non-occurence of the other event, then the events are said to be independent. Mathematically : if
P(A  B) = P(A) P(B), then A and B are independent.

Note: (i) If A and B are independent, then (a) A and B are independent, (b) A and B are independent
and (c) A and B are independent.

Probability [8]
(ii) If A and B are independent, then P(A / B) = P(A).

If events are not independent then they are said to be dependent.

Independency of three or more events


Three events A, B & C are independent if & only if all the following conditions hold :
P(A  B) = P(A) . P(B) ; P(B  C) = P(B) . P(C)
P (C  A) = P(C) . P(A) ; P(A  B  C) = P(A) . P(B) . P(C)
i.e. they must be independent in pairs as well as mutually independent.
Similarly for n events A 1, A2, A3, ........ An to be independent, the number of these conditions is equal
to n C2 + n C3 + ....... + cCn = 2n – n – 1.

Solved Example # 18
In drawing two balls from a box containing 6 red and 4 white balls without replacement, which of the
following pairs is independent ?
(a) Red on first draw and red on second draw
(b) Red on first draw and white on second draw
Solution
Let E be the event ‘Red on first draw’, F be the event ‘Red on second draw’ and G be the event ‘white
on second draw’.

6 6 4
P(E) = , P(F) = , P(G) =
10 10 10

6
P2 1
(a) P(E  F) = 10 =
P2 3

3 3 9 1
P(E) . P(F) = × = 
5 5 25 3
 E and F are not independent

6 4 6
(b) P(E) . P(G) = × =
10 10 25

6
P1 4 P1 4
P(E  G) = 10 =
P2 15

 P(E) . P(G)  P(E  G)


 E and G are not independent

Solved Example # 19
If two switches S1 and S2 have respectively 90% and 80% chances of working. Find the probabilities that
each of the following circuits will work.

Probability [9]
Solution
Consider the following events :
A = Switch S1 works,
B = Switch S2 works,
We have,
90 9 80 8
P(A) = = and P(B) = =
100 10 100 10
(i) The circuit will work if the current flows in the circuit. This is possible only when both the switches work
together. Therefore,
Required probability
= P(A  B) = P(A) P (B) [ A and B are independent events]
9 8 72 18
= × = =
10 10 100 25
(ii) The circuit will work if the current flows in the circuit. This is possible only when at least one of the two
switches S1, S2 works. Therefore,
Required Probability
= P(A  B) = 1 – P ( A ) P( B ) [ A, Bare independent events]

 9   8 
= 1 – 1   1  
 10   10 

1 2 49
= 1– × =
10 10 50

Solved Example # 20
A speaks truth in 60% of the cases and b in 90% of the cases. In what percentage of cases are they likely
to contradict each other in stating the same fact?
Solution
Let E be the event that A speaks truth and F be the event that B speaks truth. Then E and F are independent
events such that
60 3 90 9
P(E) = = and P(F) = =
100 5 100 10
A and B will contradict each other in narrating the same fact in the following mutually exclusive ways :
(i) A speaks truth and B tells a lie i.e. E  F
(ii) A tells a lie and B speaks truth lie i.e. E  F

 P(A and B contradict each other)

= P(I or II) = (I  II)


= P[(E  F )  ( E  F)]
= P(E  F ) + P ( E  F) [ E  F and E  F are mutually exclusive]
= P(E) P( F ) + P( E ) P(F) [ E and F are in dep.]

3  9   3 9 3 1 2 9 21
= × 1   + 1   × = × + × =
5  10   5 10 5 10 5 10 50

Solved Example # 21
An urn contains 7 red and 4 blue balls. Two balls are drawn at random with replacement. Find the probability
of getting
(i) 2 red balls (ii) 2 blue balls (iii) one red and one blue ball
49 16 56
Ans. (i) (ii) (iii)
121 121 121

Probability [10]
Solved Example # 22
1 1
Probabilities of solving a specific problem independently by A and B are and respectively. If both try to
2 3
solve the problem independently, find the probability that
(i) the problem is solved (ii) exactly one of them solves the problem.
2 1
Ans. (i) (ii)
3 2

Solved Example # 23
A box contains 5 bulbs of which two are defective. Test is carried on bulbs one by one untill the two defective
bulbs are found out. Find the probability that the process stops after
(i) Second test (ii) Third test
Solution
(i) Process will stop after second test. Only if the first and second bulb are both found to be defective
2 1 1
probability = × = (Obviously the bulbs drawn are not kept back.)
5 4 10
(ii) Process will stop after third test when either
2 3 1 1
(a) DND  × × = Here ‘D’ stands for defective
5 4 3 10
3 2 1 1
or (b) NDD  × × = and ‘N’ is for not defective.
5 4 3 10
3 2 1 1
or (c) NNN  × × =
5 4 3 10

3
hence required probability =
10
Solved Example # 24

1 1  E1  1
If E1 and E2 are two events such that P(E1) = ; P(E2) = ; P  
 E  = , then choose the correct options.
4 2  2 4
(i) E1 and E2 are independent (ii) E1 and E2 are exhaustive
(iii) E1 and E2 are mutually exclusive (iv) E1 & E2 are dependent
 E1   E2 
Also find P  E  and  

 2  E1 
Solution
 E2 
Since  E  = P(E1)  E1 and E2 are independent of each other
 1
Also since P(E1  E2) = P(E1) + P(E2) – P(E1) . P(E2)  1
Hence events are not exhaustive. Independent events can’t be mutually exclusive.
Hence only (i) is correct
Further since E1 & E2 are independent; E1 and E2 or E1 , E2 are E1 , E2 are also independent.

E  3 E  1
Hence P 1  = P E1 =
E  4
  and P 2  = P (E2) =
2
 2  E1 

Solved Example # 25
If cards are drawn one by one from a well shuffled pack of 52 cards without replacement,
until an ace appears, find the probability that the fourth card is the first ace to appear.

Probability [11]
Solution
48
C 3 4 C1
Probability of selecting 3 non-Ace and 1 Ace out of 52 cards is equal to 52
C4
Since we want 4th card to be first ace, we will also have to consider the arrangement, Now 4 cards in sample
space can be arranged in 4! ways and, favorable they can be arranged in 3 ! ways as we want 4th position to
be occupied by ace
48
C 3 4 C1 3!
Hence required probability = 52 × 4!
C4
Aliter :
‘NNNA’ is the arrangement than we desire in taking out cards, one by one
48 47 46 4
Hence required chance is × × ×
52 51 50 49

Self Practice Problems :

14. In throwing a pair of dies find the probability of getting an odd number on the first die and a total of
7 on both the dies.

1
Answer
12

15. In throwing of a pair of dies, find the probability of getting a boublet or a total of 4.

2
Answer
9

16. A bag contains 8 marbles of which 3 are blue and 5 are red. One marble is drawn at random, its colour is
noted and the marble is replaced in the bag. A marble is again drawn from the bag and its colour is noted.
Find the probability that the marbles will be
(i) blue followed by red (ii) blue and red in any order
(iii) of the same colour.
15 15 17
Ans. (i) (ii) (iii)
64 32 32

17. A coin is tossed thrice. In which of the following cases are the events E and F independent ?
(i) E : “the first throw results in head”.
F : “the last throw result in tail”.
(ii) E : “the number of heads is two”.
F : “the last throw result in head”.
(iii) E : “the number of heads is odd ”.
F : “the number of tails is odd”.
Ans. (i)

6. Binomial Probability Theorem

If an experiment is such that the probability of success or failure does not change with trials, then
the probability of getting exactly r success in n trials of an experiment is n Cr pr qn – r, where ‘p’ is the
probability of a success and q is the probability of a failure. Note that p + q = 1.

Solved Example 26
A pair of dice is thrown 5 times. Find the probability of getting a doublet twice.

Probability [12]
Solution

1
In a single throw of a pair of dice probability of getting a doublet is
6

1
con sidering it to be a success, p =
6

1 5
 q= 1– =
6 6
number of success r = 2
2 3
 1 5 625
 P(r = 2) = 5C2 p2 q3 = 10 .   .   =
6 6 3888

Solved Example # 27
A pair of dice is thrown 4 times. If getting ‘a total of 9’ in a single throw is considered as a success
then find the probability of getting ‘a total of 9’ thrice.
Solution

4 1
p = probability of getting ‘a total of 9’ = =
36 9

1 8
 q= 1– =
9 9
r = 3, n = 4
3
 1 8 32
 P(r = 3) = C3 p q = 4 ×  
4 3
. =
9 9 6561

Solved Example # 28
In an examination of 10 multiple choice questions (1 or more can be correct out of 4 options). A student
decides to mark the answers at random. Find the probability that he gets exactly two questions correct.
Solution
A student can mark 15 different answers to a MCQ with 4 option i.e. 4C1 + 4C2 + 4C3 + 4C4 = 15
1
Hence if he marks the answer at random, chance that his answer is correct = and being incorrecting
15
14 1 14
. Thus p= ,q= .
15 15 15
2 8
 1   14 
P (2 success) = C2 ×   ×  
10
15
   15 

Solved Example # 29
A family has three children. Event ‘A’ is that family has at most one boy, Event ‘B’ is that family has at least
one boy and one girl, Event ‘C’ is that the family has at most one girl. Find whether events ‘A’ and ‘B’ are
independent. Also find whether A, B, C are independent or not.
Solution
A family of three children can have
(i) All 3 boys (ii) 2 boys + 1 girl (iii) 1 boy + 2 girls (iv) 3 girls
3
 1 1
(i) P (3 boys) = C0   =
3
(Since each child is equally likely to be a boy or a girl)
2
  8

Probability [13]
2
 1 1 3
(ii) P (2 boys +1girl) = 3C1 ×   × = (Note that there are three cases BBG, BGB, GBB)
2 2 8
1 2
 1  1 3
(iii) P (1 boy + 2 girls) = 3C2 ×   ×   =
 2 2 8

1
(iv) P (3 girls) =
8
1
Event ‘A’ is associated with (iii) & (iv). Hence P(A) =
2
3
Event ‘B’ is associated with (ii) & (iii). Hence P(B) =
4
1
Event ‘C’ is associated with (i) & (ii). Hence P(C) =
2
3
P(A B) = P(iii) = = P(A) . P(B) . Hence A and B are independent of each other
8
P(A C) = 0  P(A) . P(C) . Hence A, B, C are not independent

Self Practice Problems :

18. A box contains 2 red and 3 blue balls. Two balls are drawn successively without replacement. If getting
‘a red ball on first draw and a blue ball on second draw’ is considered a success, then find the probability
of 2 successes in 3 performances.
Answer .189

19. Probability that a bulb produced by a factory will fuse after an year of use is 0.2. Find the probability
that out of 5 such bulbs not more than 1 bulb will fuse after an year of use.

2304
Answer
3125

7. Expectation :
If a value M i is associated with a probability of p i , then the expectation is given by  piMi.

Solved Example # 30
There are 100 tickets in a raffle (Lottery). There is 1 prize each of Rs. 1000/-, Rs. 500/- and
Rs. 200/-. Remaining tickets are blank. Find the expected price of one such ticket.
Solution
Expectation =  piMi outcome of a ticket can be
pi Mi piMi

1
(i) I prize 1000 10
100
1
(ii) II prize 500 5
100
1
(iii) III prize 200 2
100
97
(iv) Blank 0 0
100

Probability [14]
________________
 piMi = 17
________________
Hence expected price of one such ticket Rs. 17

Solved Example # 31
A purse contains four coins each of which is either a rupee or two rupees coin. Find the expected value of a
coin in that purse.
Solution
Various possibilities of coins in the purse can be
pi Mi piMi

1 4
(i) 4 1 rupee coins 4
16 16
4 20
(ii) 3 one Rs. + 1 two Rs. 5
16 16
6 36
(iii) 2 one Rs. + 2 two Rs. 6
16 16
4 28
(iv) 1 one Rs. + 3 two Rs. 7
16 16
1 8
(iv) 4 two Rs. 8
16 16
________________
6/-
________________
Note that since each coin is equally likely to be one Rs. or two Rs. coin, the probability is determined using
Binomial probability; unlike the case when the purse contained the coins with all possibility being equally
1
likely, where we take pi = for each.
5
Hence expected value is Rs. 6/-

Self Practice Problems :

20. From a bag containing 2 one rupee and 3 two rupee coins a person is allowed to draw 2 coins indiscrimi-
nately; find the value of his expectation.
Ans. Rs. 3.20

8. Total Probability Theorem


If an event A can occur with one of the n mutually exclusive and exhaustive events B 1, B2, ....., Bn
and the probabilities P(A/B 1), P(A/B2) .... P(A/Bn ) are known, then

n
P(A) =  P(B ) . P(A / B )
i 1
i i

Solved Example # 32

Box -  contains 5 red and 4 white balls whilst box -  contains 4 red and 2 white balls. A fair die is
thrown. If it turns up a multiple of 3, a ball is drawn from box -  else a ball is drawn from box - . Find
the probability that the ball drawn is white.
Solution
Let A be the event ‘a multiple of 3 turns up on the die’ and R be the event ‘the ball drawn is white’
then P (ball drawn is white)

Probability [15]
= P(A) . P(R / A) + P ( A ) P(R / A )

2 4  2 2 10
= × + 1   =
6 9  6 6 27

Solved Example # 33

Cards of an ordinary deck of playing cards are placed into two heaps. Heap - consists of all the red
cards and heap - consists of all the black cards. A heap is chosen at random and a card is drawn,
find the probability that the card drawn is a king.
Solution
Let  and be the events that heap -  and heap - are choosen respectively. Then

1
P() = P() =
2
Let K be the event ‘the card drawn is a king’

2 2
 P (K /) = and P(K /) =
26 26

1 2 1 2 1
 P(K) = P () P(K / ) + P() P(K /) = × + × = .
2 26 2 26 13

Self Practice Problems :

21. Box -  contains 3 red and 2 blue balls whilest box - II contains 2 red and 3 blue balls. A fair coin is
t ossed. I f i t t urns up head, a bal l i s drawn f rom box - , el se a bal l i s drawn f rom
box -  . Find the probability that the ball drawn is red.

1
Answer
2

22. There are 5 brilliant students in class XI and 8 brilliant students in class XII. Each class has 50
students. The odds in favour of choosing the class XI are 2 : 3. If the class XI is not chosen then the
class XII is chosen. Find the probability of selecting a brilliant student.

17
Answer .
125

9. Bayes’ Theorem :
If an event A can occur with one of the n mutually exclusive and exhaustive events B 1, B2 , ....., Bn and
the probabilities P(A/B 1), P(A/B2) .... P(A/Bn ) are known, then

P(B i ) . P( A / B i )
P(Bi / A) = n

 P(B ) . P( A / B )
i 1
i i

Proof :
The event A occurs with one of the n mutually exclusive and exhaustive events
B1, B2, B3,........,Bn
A = (A  B1)  (A  B2)  (A  B3)  ........  (A  Bn )

Probability [16]
n

P(A) = P(A  B1) + P(A B2) + ....... + P(A Bn ) =  P(A  B )


i1
i

Note: A  event what we have ; Bi = event what we want ;


Now,
P(A  Bi) = P(A) . P(Bi/A) = P(Bi) . P(A/Bi)

P(Bi ) . P( A / Bi ) P(B i ) . P( A / B i )
P (Bi/A) = = n
P( A )

P( A  B i )
i 1

P(Bi ) . P( A / Bi )
P(Bi/A) =
 P(B ) . P( A / B )
i i

Solved Example # 34
2
Pal’s gardener is not dependable, the probability that he will forget to water the rose bush is . The rose
3
1
bush is in questionable condition any how, if watered the probability of its withering is , if not watered, the
2
3
probability of its withering is. Pal went out of station and upon returning, he finds that the rose bush has
4
withered, what is the probability that the gardener did not water the bush.
[Here result is known that the rose bush has withered, therefore. Bayes’s theorem should be used]
Solution
Let A = the event that the rose bush has withered
Let A1 = the event that the gardener did not water.
A2 = the event that the gardener watered.
By Bayes’s theorem required probability,
P( A 1 ) . P( A / A 1 )
P(A1/A) = P( A ) . P( A / A )  P( A ) . P( A / A ) .....(i)
1 1 2 2

2 1
Given, P(A1) =  P(A2) =
3 3
3 1
P(A/A1) = , P(A/A2) =
4 2

2 3
.
3 4 6 3
From (1), P(A1/A) = 2 3 1 1 = =
.  . 62 4
3 4 3 2

Solved Example # 35
There are 5 brilliant students in class XI and 8 brilliant students in class XII. Each class has 50
students. The odds in favour of choosing the class XI are 2 : 3. If the class XI is not chosen then the
Probability [17]
class XII is chosen. A student is a chosen and is found to be brilliant, find the probability that the
chosen student is from class XI.
Solution
Let E and F be the events ‘Class XI is chosen’ and ‘Class XII is chosen’ respectively.

2 3
Then P(E) = , P(F) =
5 5
Let A be the event ‘Student chosen is brilliant’.

5 8
Then P(A / E) = and P(A / F) = .
50 50

2 5 3 8 34
 P(A) = P(E) . P(A / E) + P(F) . P(A / F) = . + . = .
5 50 5 50 250

P(E) . P( A / E) 5
 P(E / A) = P(E) . P( A / E)  P(F) . P( A / F) = .
17

Solved Example # 36
A pack of cards is counted with face downwards. It is found that one card is missing. One card is drawn and
is found to be red. Find the probability that the missing card is red.
Solution
Let A be the event of drawing a red card when one card is drawn out of 51 cards (excluding missing card.) Let
A1 be the event that the missing card is red and A2 be the event that the missing card is black.
Now by Bayes’s theorem, required probability,
P( A 1 ) . (P( A / A1 )
P(A1/A) = P( A ) . P( A / A )  P( A ) . P( A / A ) ..........(i)
1 1 2 2
In a pack of 52 cards 26 are red and 26 are black.
26
C1 26 1
Now P(A1) = probability that the missing card is red = 52 = =
C1 52 2

26 1
P(A2) = probability that the missing card is black = =
52 2
P(A/A1) = probability of drawing a red card when the missing card is red.
25
=
51
[ Total number of cards left is 51 out of which 25 are red and 26 are black as the missing card is red]
26
Again P(A/A2) = Probability of drawing a red card when the missing card is black =
51
Now from (i), required probability,
1 25
.
2 51 25
P(A1/A) = 1 25 1 26 =
.  . 51
2 51 2 51

Solved Example # 37
A bag contains 6 white and an unknown number of black balls ( 3). Balls are drawn one by one with
replacement from this bag twice and is found to be white on both occassion. Find the probability that the bag
had exactly ‘3’ Black balls.

Probability [18]
Solution
Apriori, we can think of the following possibilies
(i) E1 6W , 0B
(ii) E2 6W , 1B
(iii) E3 6W , 2B
(iv) E4 6W , 3B
1
Clearly P(E1) = P(E2) = P(E3) = P(E4) =
4
Let ‘A’ be the event that two balls drawn one by one with replacement are both white therefore we have to find
 E4 
P  
 A 

  A
P
  P(E 4 )
 E4    E4
By Baye’s theorem P   =
 A   A   A   A   A 
P   P(E1 )  P  . P(E 2 )  P  . P(E 3 )  P  . P(E 4 )
 E1   E2   E3   E4 
 A  6 6  A  6 6  A  6 6
P  E  = × ; P  E  = × ; P  E  = × ;
 4 9 9  3 8 8  2 7 7

A 6 6
P  E  = × ;
 1 6 6

1
 E4  81
Putting values P   =
 A  1 1 1 1
  
81 64 49 36

Self Practice Problems :

23. Box- contains 3 red and 2 blue balls whilest box- contains 2 red and 3 blue balls. A fair coin is tossed.
If it turns up head, a ball is drawn from box-, else a ball is drawn from box-. If the ball drawn is red,
then find the probability that the ball is drawn from box-.

3
Answer
5

24. Cards of an ordinary deck of playing cards are placed into two heaps. Heap - consists of all the red
cards and heap - consists of all the black cards. A heap is chosen at random and a card is drawn,
if the card drawn is found to be a king, find the probability that the card drawn is from the heap - .

1
Answer
2

10. Value of Testimony

If p1 and p2 are the probabilities of speaking the truth of two independent witnesses A and B then P(their

p1 p 2
combined statement is true) = .
p1 p 2  (1  p1 )(1  p 2 )

In this case it has been assumed that we have no knowledge of the event except the statement made
by A and B.
However if p is the probability of the happening of the ev ent before their statement, then

Probability [19]
p p1 p 2
P(their combined statement is true) = .
p p1 p 2  (1  p) (1  p1 )(1  p 2 )

Here it has been assumed that the statement given by all the independent witnesses can be given in
two ways only, so that if all the witnesses tell falsehoods they agree in telling the same falsehood.
If this is not the case and c is the chance of their coincidence testimony then the
Probability that the statement is true = P p 1 p2
Probability that the statement is false = (1 – p). c (1 – p 1) (1 – p2)
However chance of coincidence testimony is taken only if the joint statement is not contradicted by
any witness.

Solved Example # 38
A die is thrown, a man C gets a prize of Rs. 5 if the die turns up 1 and gets a prize of Rs. 3 if the

1
die turns up 2, else he gets nothing. A man A whose probability of speaking the truth is tells C that
2

2
the die has turned up 1 and another man B whose probability of speaking the truth is tells C that
3
the die has turned up 2. Find the expectation value of C.
Solution

1
Let A and B be the events ‘A speaks the truth’ and ‘B speaks the truth’ respectively. Then P(A) =
2

2
and P(B) = .
3
The experiment consists of three hypothesis
(i) the die turns up 1
(ii) the die turns up 2
(iii) the die turns up 3, 4, 5 or 6

1 4
Let these be the events E 1 , E2 and E 3 respectively then P(E 1) = P(E 2) = and P(E 3) = .
6 6
Let E be the event that the statements made by A and B agree to the same conclusion.

1 1 1
 P(E / E1) = P(A) . P (B ) = × =
2 3 6

1 2 2
P(E / E2) = P( A ) . P(B) = × =
2 3 6

1 1 1
P(E / E3) = P( A ) . P (B ) = × =
2 3 6
 P(E) = P(E1) P(E / E1) + P(E2) P(E / E2) + P(E3) P(E / E3)

1 1 1 2 4 1 7
= . + . + . =
6 6 6 6 6 6 36

P(E1 )P(E / E1 ) 1
Thus P(E1 / E) = =
P(E) 7

Probability [20]
P(E2 )P(E / E2 ) 2
P(E2 / E) = =
P(E) 7

P(E3 )P(E / E3 ) 4
P(E3 / E) = =
P(E) 7

1 2 11
 expectation of C = ×5+ × 3 + 0 = Rs.
7 7 7

Solved Example #39


A speaks the truth ‘3 times out of 4’ and B speaks the truth ‘2 times out of 3’. A die is thrown. Both
assert that the number turned up is 2. Find the probability of the truth of their assertion.
Solution
Let A and B be the events ‘A speaks the truth’ and ‘B speaks the truth’ repectively. Let C be the event
‘the number turned up is not 2 but both agree to the same conclustion that the die has turned up 2’.

3 2 1 1
Then P(A) = , P(B) = and P(C) = ×
4 3 5 5
There are two hypotheses
(i) the die turns up 2
(ii) the die does not turns up 2
Let these be the events E 1 and E2 respectively, then

1 5
P(E1) = , P(E2) = (a priori probabilities)
6 6
Now let E be the event ‘the statement made by A and B agree to the same conclusion.

3 2 1
then P(E / E1) = P(A) . P(B) = . =
4 3 2

1 1 1 1
P(E / E2) = P( A ) . P( B ) . P(C) = . . =
4 3 25 300
Thus P(E) = P(E1) P(E / E1) + P(E2) P(E / E2)

1 1 5 1 31
= × + × =
6 2 6 300 360

P(E1 ) P(E / E1 ) 30
 P(E1 / E) = =
P(E) 31

Self Practice Problems :

25. A ball is drawn from an urn containing 5 balls of different colours including white. Two men A and B

1 2
whose probability of speaking the truth are and respectively assert that the ball drawn is white.
3 5
Find the probability of the truth of their assertion.

4
Answer
7

Probability [21]
11. Binomial Probability Distribution :

(i) A probability distribution spells out how a total probability of 1 is distributed over several values of a
random variable.
(ii) Mean of any probability distribution of a random variable is given by :

 pi x i
µ= =  pi x i (Since  pi = 1)
 pi

(iii) Variance of a random variable is given by, 2 = (x i – µ)2 . pi


 2 = pi x i2 – µ2 (Note that SD = +  2 )
(iv) The probability distribution for a binomial variate ‘X’ is given by :
P(X = r) = n Cr pr qn – r where P(X = r) is the probability of r successes.

P(r  1) nr p
The recurrence formula = . , is very helpful for quickly computing P(1) . P(2) . P(3)
P(r ) r 1 q
etc. if P(0) is known.
(v) Mean of BPD = np ; variance of BPD = npq.
(vi) If p represents a person’s chance of success in any venture and ‘M’ the sum of money which he will
receive in case of success, then his expectations or probable value = pM

Solved Example # 40
A random variable X has the following probability distribution :

X 0 1 2 3 4 5 6 7
2 2 2
P(X) 0 k 2k 2k 3k k 2k 7k + k
Determine
(i) k (ii) P(X < 3) (iii) P(X > 6) (iv) P(0 < X < 3)
[Hint : Use  P(X) = 1 to determine k, P(X < 3) = P(0) + P(1) + P(2), P(X > 6) = P(7) etc.]

Solved Example # 41
A pair of dice is thrown 5 times. If getting a doublet is considered as a success, then find the mean
and variance of successes.
Solution

1
In a single throw of a pair of dice, probability of getting a doublet =
6

1
con sidering it to be a success, p =
6

1 5
 q= 1– =
6 6

1 5
mean = 5 × =
6 6

1 5 25
variance = 5 × . =
6 6 36

Solved Example # 42
A pair of dice is thrown 4 times. If getting a total of 9 in a single throw is considered as a success
then find the mean and variance of successes.
Probability [22]
Solution

4 1
p = probability of getting a total of 9 = =
36 9

1 8
 q= 1– =
9 9

1 4
 mean = np = 4 × =
9 9

1 8 32
variance = npq = 4 × × =
9 9 81

Solved Example # 43
Difference between mean and variance of a Binomial variate is ‘1’ and difference between their squares is ‘11’.
Find the probability of getting exactly three success
Solution
Mean = np & variance = npq
therefore, np – npq = 1 ..........(i)
n2p2 – n2p2q2 = 11 ..........(ii)
Also, we know that p + q = 1 ..........(iii)
5 1
Divide equation (ii) by square of (i) and solve, we get, q = ,p= & n = 36
6 6
3 33
 1 5
Hence probability of ‘3’ success = C3 ×   ×  
36
Ans.
6 6

Self Practice Problems :

26. A box contains 2 red and 3 blue balls. Two balls are drawn successively without replacement. If getting
‘a red ball on first draw and a blue ball on second draw’ is considered a success, then find the mean
and variance of successes.
Answer mean = 2.1, 2 = .63

27. Probability that a bulb produced by a factory will fuse after an year of use is 0.2. If fusing of a bulb
is considered an failure, find the mean and variance of successes for a sample of 10 bulbs.
Answer mean = 8 and variance = 1.6

28. A random variable X is specified by the following distribution law :

X 2 3 4
P(X = x) 0.3 0.4 0.3
Then the variance of this distribution is :
(A*) 0.6 (B) 0.7 (C) 0.77 (D) 1.55

12. Geometrical Applications:


The following statements are axiomatic :

(i) If a point is taken at random on a given straight line segment AB, the chance that it falls on a particular
segment PQ of the line segment is PQ/AB.
(ii) If a point is taken at random on the area S which includes an area , the chance that the point falls
on  is /S.
Probability [23]
Solved Example # 44
A sphere is circumscribed over a cube. Find the probability that a point lies inside the sphere, lies outside the
cube.
Solution

favorable volume
Required probability =
total volume

a 3
Clearly if edge length of cube is a radius of sphere will be
2
3
4  a 3  3
 = a 3
Thus, volume of sphere = 
3  2 
 2

1 2
Hence P = 1 – =1–
3  3

2

Solved Example # 45
A given line segment is divided at random into three parts. What is the probability that they form sides
of a possible triangle ?
Solution
Let AB be the line segment of length .
Let C and D be the points which divide AB into three parts.
Let AC = x, CD = y. Then DB =  – x – y.
Clearly x + y < 
 the sample space is given by
the region enclosed by  OPQ, where OP = OQ = 

2
Area of OPQ =
2
Now if the parts AC, CD and DB form a triangle, then


x + y >  –x – y i.e. x + y > ...........(i)
2


x + – x – y > y i.e. y < ...........(ii)
2


y + – x – y > x i.e. x < ...........(iii)
2
from (i), (ii) and (iii), we get
the event is given by the region closed in RST

Probability [24]
1  
ar ( RST ) . .
2 2 2 1
 Probability of the event = ar ( OPQ ) = 2 =
 4
2

Solved Example # 46
On a line segment of length L two points are taken at random, find the probability that the distance
between them is  , where  < 1
Solution
Let AB be the line segment
Let C and D be any two points on AB so that AC = x and CD = y. Then x + y < L, y > 
 sample space is represented by the region enclosed by OPQ.

1 2
Area of OPQ = L
2

The event is represented by the region, bounded by the RSQ

1
Area of RSQ = (L – )2
2

2
L 
 probability of the event =  
 L 

Self Practice Problems :

29. A line segment of length a is divided in two parts at random by taking a point on it, find the probability
that no part is greater than b, where 2b > a

2b  a
Answer
a
30. A cloth of length 10 meters is to be randomly distributed among three brothers, find the probability that
no one gets more than 4 meters of cloth.

1
Answer
25

Probability [25]
LEVEL # 1
Q.1 The probability P (A) of an event is a -
(A) real number (B) positive real number
(C) non- negative real number (D) non- negative real number  1

Q.2 Winning a game by a player is-


(A) an experiment (B) an event
(C) experiment and event both (D) None of these

Q.3 In tossing a coin getting a head or tail is -


(A) experiment (B) exclusive event (C) joint event (D) None of these

Q.4 The correct statement for any event A is -


(A) 0  P(A)  1 (B) 0  P(A)  1 (C) P (A) = 1 (D) P(A) < 0

Q.5 A coin is tossed three times. The probability that in the second toss head does not occur, is-
(A) 1 (B) 1/2 (C) 1/3 (D) 1/4

Q.6 The probability of coming up an even (odd) number in the throw of a die is-
(A) 1/6 (B) 1/2 (C) 1/3 (D) 2/3

Q.7 From a pack of playing cards three cards are drawn simultaneously. The probability that these are one
king, one queen and one jack is-
(A) 64/5525 (B) 16/5525 (C) 128/5525 (D) 64/525

Q.8 The probability of getting difference of number as 5, when two dice are tossed together is-
(A) 1/9 (B) 1/18 (C) 1/12 (D) 5/36

Q.9 The probability of drawing a black king from a pack of 52 cards is-
(A) 1/13 (B) 1/26 (C) 2/13 (D) 4/13
Q.10 Three cards are drawn from a pack of 52 cards. The probability that they are of the same colour is-
(A) 4/17 (B) 22/225 (C) 3/17 (D) 2/17

Q.11 A bag contains 6 blue, 4 white and 6 red balls. Two balls are drawn at random. The probability that
both the balls are red is-
(A) 1/3 (B) 1/6 (C) 1/8 (D) 2/9

Q.12 A bag contains 20 tickets numbered with 1 to 20. Three tickets are drawn. The probability that ticket
number 7 is definitely included and ticket number 18 is not included is-
(A) 51/380 (B) 1/20 (C) 3/20 (D) None of these

Q.13 From a lottery of 30 tickets, marked 1, 2, 3,...., 30, four tickets are drawn. The chance that those marked
1 and 2 are among them is-
(A) 413/145 (B) 2/145 (C) 1/145 (D) 4/145
Q.14 The probability that a non leap year will have 53 Saturdays is-
(A) 1/7 (B) 2/7 (C) 6/7 (D) 5/7
Q.15 The probability that a non leap year will have 52 Fridays is-
(A) 1/7 (B) 2/7 (C) 5/7 (D) 6/7

Q.16 The probability that a leap year will have 52 Sundays is-
(A) 1 (B) 5/7 (C) 2/7 (D) None of these
Probability [26]
Q.17 There are 13 men and 2 women in a party. They are seated round a circular table. The probability that the
two women will sit together is-
(A) 2/105 (B) 1/105 (C) 1/14 (D) 1/7

Q.18 A bag contains two pairs of shoes. Two shoes are drawn from it. The probability that it is a pair is-
(A) 1/3 (B) 1/2 (C) 1/4 (D) 2/3
Q.19 If out of 20 consecutive whole numbers two are chosen at random, then the probability that their sum
is odd, is-
(A) 5/19 (B) 10/19 (C) 9/19 (D) None of these

Q.20 If the probabilities of boy and girl to be born are same, then in a 4 children family the probability of
being at least one girl, is-
(A) 14/16 (B) 15/16 (C) 1/8 (D) 3/8

Q.21 If 4 cards are drawn one by one from a pack of 52 cards, the probability that one will be from each
suit, is-
13 13 13 13 13 13 13 13
(A) × × × (B) × × × ×24
52 39 26 13 52 51 50 49

13 13 13 13 13 13 13 13
(C) × × × ×24 (D) × × ×
52 39 26 13 52 51 50 49

Q.22 The probability that two persons have same date of birth is (in non-leap year)
(A) 0 (B) 1 (C) 1/365 (D) 364/365

Q.23 Two coins are tossed together. The probability of getting two heads is-
(A) 1/2 (B) 1/4 (C) 1/8 (D) 1/3

Q.24 Two dice are thrown together. The probability that the sum of their two numbers be 10 is-
(A) 1/6 (B) 1/12 (C) 2/3 (D) 1/4

Q.25 An urn contains 5 white and 3 black balls and 4 balls are drawn at random. The probability of getting
white and black balls equal in number is-
(A) 1/7 (B) 2/7 (C) 3/7 (D) None of these

Q.26 From a book containing 100 page one page is selected randomly. The probability that the sum of the
digits of the page number of the selected page is 11, is-
(A) 2/25 (B) 9/100 (C) 11/100 (D) None of these

Q.27 A bag contains 20 tickets marked with numbers 1 to 20. Two tickets are drawn, the probability that
both numbers are prime is-
(A) 4/95 (B) 7/95 (C) 14/95 (D) 1/10
Q.28 Two dice are thrown, the probability that the total score is a prime number is-
(A) 1/6 (B) 5/12 (C) 1/2 (D) None of these

Q.29 A box contains 25 tickets numbered 1, 2,....25. If two tickets are drawn at random then the probability
that the product of their numbers is even, is -
(A) 11/50 (B) 13/50 (C) 37/50 (D) None of these

Q.30 A bag contains 8 white and 6 red balls. 5 balls are drawn from the bag at random. The probability that 3 or
more balls are white will be-
(A) 658/1001 (B) 317/1001 (C) 205/1001 (D) 210/1001

Probability [27]
Q.31 Out of 13 applicants for a job, there are 5 women and 8 men. It is desired to select 2 persons for
the job, the probability that at least one of the selected persons will be a women is-
(A) 25/39 (B) 14/39 (C) 5/13 (D) 10/13
Q.32 The odd against throwing 10 with two dice in a throw are-
(A) 9 : 1 (B) 10 : 1 (C) 11 : 1 (D) 12 : 1

Q.33 From a pack of well shuffled cards, one card is drawn randomly. A gambler bets that it is either a
diamond or a king. The odds in favour of his winning the bet is-
(A) 4 : 7 (B) 4 : 9 (C) 9 : 4 (D) None of these

Q.34 A bag contains 4 red and 4 white balls. Three balls are drawn at random. The odd against these balls
being all white are-
(A) 1 :13 (B) 13 : 1 (C) 12 : 1 (D) 14 : 1

Q.35 One of the two events must occur. If the chance of one is 2/3 of the other, then odds in favour of the
other are-
(A) 2 : 3 (B) 1 : 3 (C) 3 : 1 (D) 3 : 2

Q.36 If odds in favour of A is m : n then P(A) equals-


(A) m/n (B) m – n (C) m / (m + n) (D) m / (m – n)

Q.37 A dice is thrown then the odds against of getting the digit 6 is-
(A) 5 : 1 (B) 4 : 1 (C) 6 : 1 (D) None of these
Q.38 If one card is drawn from a pack of card then the odds in favour of getting Ace is-
(A) 1 : 3 (B) 3 : 1 (C) 1 : 2 (D) 1 : 12

Q.39 A bag contains 3 black and 2 white balls. What are the odds in favour of drawing a white ball?
(A) 3 : 2 (B) 2 : 5 (C) 2 : 3 (D) 3 : 5

Q.40 If A and B be any two events, then P (A  B) equals-


(A) P(A) – P (B) (B) P (A) + P(B) + P (A  B)
(C) P (A) + P(B) – P (A  B) (D) P (A) P (B)
Q.41 P (A + B) means-
(A) Probability of event A and B (B) Probability of event A or B
(C) Probability of event B after happening of event A (D) None of these
Q.42 If P (A) + P (B) = P (A + B) then events A & B are-
(A) Independent (B) Mutually exclusive (C) Dependent (D) None of these
Q.43 If A and B are two events, then P (neither A nor B) equals-
(A) 1 – P (A  B) (B) P ( A ) + P ( B )
(C) 1 – P (A) – P (B) (D) None of these

Q.44 For any two events A and B, P (A+B)-


(A) is always equal to P (A) + P (B) (B) never equals to P (A) + P (B)
(C) equals P(A) + P(B) If A and B are independent (D) equals P (A) + P (B) If A and B are disjoint

Q.45 If two dice are thrown, then the probability of getting the sum of digits even or less than 5 is-
(A) 1/2 (B) 1/6 (C) 2/3 (D) 5/9

Q.46 Two dice are thrown together. The probability that the sum of their numbers be at least 5 is-
(A) 1/6 (B) 5/6 (C) 4/6 (D) None of these

Q.47 Two dice are thrown then the probability of coming an odd number on one dice and an even number on
other dice is-
(A) 1/3 (B) 1/6 (C) 1/2 (D) None of these
Probability [28]
Q.48 A card is drawn from a pack of 52 cards. The probability that the card drawn is neither a heart nor
a king is-
(A) 35/52 (B) 9/13 (C) 17/52 (D) 4/13
Q.49 A bag is containing 20 balls, which are arranged in order of their numbers. If one ball is drawn at
random, then the probability that its number is multiple of 3 or 5 is-
(A) 9/20 (B) 1/20 (C) 3/50 (D) None of these

Q.50 The odds in favour of winning a race by three boys are 1:3, 1:5 and 1:4 respectively. The probability
of winning the race by only one of them is-
(A) 7/20 (B) 47/120 (C) 17/60 (D)37/60

Q.51 If P(A) = 0.25, P(B) = 0.50 and P (A  B) = 0.14, then the probability that neither A nor B occurs is-
(A) 0.39 (B) 0.25 (C) 0.11 (D) None of these

3
Q.52 A, B and C are three mutually exclusiv e and exhaustiv e ev ents and P(B) = P (A),
2
1
P(C) = P(B) then the value of P (A) is-
3
(A) 1/3 (B) 1/2 (C) 1/6 (D) None of these

Q.53 A card is drawn from a well shuffled pack of 52 cards. Its probability of being an ace or a king or
a queen or a jack is -
(A) 1/13 (B) 2/13 (C) 3/13 (D) 4/13

Q.54 Let P (A) = 0.4 and P (B/A) = 0.5. The probability P ( A  B ) is equal to-
(A) 0.8 (B) 0.7 (C) 0.6 (D) None of these

Q.55 A pair of dice is thrown. If 5 appears on at least one of the dice, then the probability that the sum
is 10 or greater, is-
(A) 11/36 (B) 2/9 (C) 3/11 (D) 1/12
Q.56 A pair of dice is thrown. If the two numbers appearing on them are different, the probability that the
sum is 6, is-
(A) 2/15 (B) 1/9 (C) 5/36 (D) 1/12

Q.57 Two dice are thrown together. If 3 appears on at least one of the dice, then what is the probability
that the sum is greater than 9-
(A) 1/4 (B) 3/11 (C) 5/11 (D) zero
Q.58 In a certain town, 40% of the people have brown hair, 25% have brown eyes and 15% have both brown
hair and brown eyes. If a person selected at random has brown hair, the probability that he also has brown
eyes is-
(A) 2/5 (B) 1/4 (C) 1/2 (D) 3/8
Q.59 A bag contains 7 red and 3 black balls. Three balls are drawn at random from the bag one after the
other. The probability that the first two are red and the third is black is-
(A) 21/40 (B) 1/5 (C) 7/50 (D) 7/40
Q.60 If A and B are two independent events then P (A  B) equals-
(A) P(A) + P (B) (B) P (A) . P (B) (C) P (A/B) (D) None of these
Q.61 If A and B are two independent events, then the probability that only one of A and B occur is-
(A) P (A) + P (B) – 2P (A  B) (B) P (A) + P(B) – P (A  B)
(C) P (A) + P (B) (D) None of these

Q.62 For two given events A and B, the relation P (AB) = P (A) P(B) implies that A and B are-
(A) Independent (B) Mutually exclusive (C) Dependent (D) None of these

Probability [29]
Q.63 If p1 and p2 are the probabilities of two independent events then (1 – p1 – p2 + p1p2) is the probability
of -
(A) Their joint occurrence (B) Occurrence of at least one
(C) Occurrence of None of these (D) Occurrence of only one
Q.64 A coin is tossed three times. The probability of getting all heads or tails only is-
(A) 0 (B) 1/2 (C) 1/4 (D) 1

Q.65 A coin is tossed four times then the probability of obtaining at least one tail is -
(A) 1/16 (B) 14/16 (C) 15/16 (D) 1/4

Q.66 The probability of getting head and tail alternatively in three throws of a coin (or in a throw of three
coins) is-
(A) 1/3 (B) 1/4 (C) 1/5 (D) 3/5

Q.67 The probability of not getting tail in the first two times and getting a tail in the third time by tossing
a coin continuously is-
(A) 1/4 (B) 1/8 (C) 3/8 (D) 7/8

Q.68 From a pack of 52 cards two cards are drawn in succession the first having been replaced before the
second is drawn. The probability that the first is a diamond and the second is a king, is-
(A) 1/52 (B) 1/13 (C) 1/4 (D) 4/13

Q.69 If two cards are drawn from a pack of card one by one. If first drawn card is replaced then the
probability of getting two jacks is-
(A) 1/221 (B) 1/169 (C) 12/221 (D)4/663
Q.70 A bag contains 6 black and 5 white balls, while the second bag contains 7 black and 4 white balls.
Two balls are drawn one from each bag, the probability of both being black is-
(A) 2/11 (B) 11/13 (C) 20/121 (D)42/121

Q.71 The probability that A will pass in a examination is 2/5 and the probability that B will fail in the same
examination is 3/4. The probability that only one of them will pass in the examination is-
(A) 3/20 (B) 3/10 (C) 9/20 (D) None of these

Q.72 For solving a problem, odds against to A are 4 : 3 and odds in favour to B are 7: 5. The probability
that the problem will not be solved is-
(A) 16/21 (B) 5/21 (C) 43/84 (D)45/84

Q.73 The probability of solving a problem by A and B are 1/4 and 2/3 respectively. If A and B work independently,
then the probability that the problem will be solved by both of them is-
(A) 1/6 (B) 3/4 (C) 1/3 (D)11/12

Q.74 The probabilities that three boys will pass an examination are 1/6, 1/4 and 1/3 respectively.
The probability that exactly one boy will pass the examination is -
(A) 31/72 (B) 7/12 (C) 41/72 (D) 11/12

Q.75 If A and B are any two events such that P (A + B) = 5/6, P (AB) = 1/3, P ( B ) = 1/2, then the eventss
A and B are-
(A) independent (B) dependent (C) mutually exclusive (D) exhaustive
Q.76 A card is drawn from a pack of playing cards. It is replaced in the pack and the pack is shuffled, and
again a card is drawn. This process as repeated six times, then probability of getting in sequence
2 heart, 2 diamond and 2 black cards is -
(A) (1/4)4 (B) (1/4)5 (C) (1/4)6 (D) None of these
Q.77 A man and a woman appear in an interview for two vacancies in the same post. The probability of man’s
selection is 1/4 and that of the woman’s selection is 1/3. What is the probability that none of them
will be selected-
(A) 1/2 (B) 1/12 (C) 1/4 (D) None of these
Probability [30]
Q.78 If the probabilities of three persons A, B & C hitting a target are 3/5, 2/5 and 3/4 respectively. If they
hit at a time then the probability that two persons hit the target is-
(A) 9/50 (B) 9/20 (C) 11/20 (D) 41/50

Q.79 A bag contains 4 black and 3 white balls. Two- two balls are drawn two times. If balls are not replace
once it is drawn then the probability that first two balls are black and second two balls are white is-
(A) 4/49 (B) 2/35 (C) 1/35 (D) 3/35

Q.80 The probability that Krishna will be alive 10 years hence is 7/15 and Hari will be alive is 7/10. The
probability that both Krishna and Hari will be dead 10 years hence is-
(A) 21/150 (B) 24/150 (C) 49/150 (D) 56/150

Q.81 From the records of a hospital, it is found that 20% patients died with the disease cancer. If two
patients with cancer are admitted to hospital; then probability that at least one patient will be cured,
is-
(A) 16/25 (B) 24/25 (C) 9/25 (D) None of these

Q.82 A draws two cards one by one (replacing previous one) from a pack of cards and B throws two dice
together. The probability that both cards of A are of the same suit and the sum of digits of B is 6, will
be-
(A) 1/4 (B) 1/44 (C) 5/144 (D)7/144

Q.83 India plays two matches each with West Indies and Australia. In any match the probability to get
0,1 and 2 point by India are 0.45, 0.05 and 0.50 respectively. If the results are independent, then the
probability that India gets at least 7 points is-
(A) 0.8750 (B) 0.0875 (C) 0.6250 (D) 0.0250
Q.84 The probability that a man will remain alive for the next 25 years is 4/5 and the probability that his
wife will remain alive for the same 25 years is 3/4. The probability that at least one of them will be
alive 25 years hence, is-
(A) 19/20 (B) 3/5 (C) 3/20 (D) None of these

Q.85 A piece of equipment will function only when all the three components A, B and C are working. the
probability of A failing during one year is 0.15, that of B is 0.05 and that of C is 0.10. The probability
that the equipment will fail before the end of the year is-
(A) 0.72675 (B) 0.27325 (C) 1 (D) 0.95

Q.86 A box contains 15 tickets numbered 1, 2,...., 15. Seven tickets are drawn at random one after the other
with replacement. The probability that every time the greatest number on a drawn ticket is 9, is-
(A) (9/10)6 (B) (8/15)7 (C) (3/5)7 (D) None of these

Q.87 The probability that an event A happens in one trial of an experiment is 0.4. Three independent trails
of the experiment are performed. The probability that the event A happens at least once is-
(A) 0.936 (B) 0.784 (C) 0.904 (D) None of these

Q.88 A pair of dice is thrown four times. If getting the same number on both dice is considered as a
success, the probability of getting two success is-
(A) 20/216 (B) 25/216 (C) 19/216 (D) None of these

Q.89 A cube is thrown 6 times, then probability of getting the digits 2 and 4 exactly three times each is-
(A) 1/5184 (B) 5/11664 (C) 1/46656 (D) 3/11664
Q.90 A box of 100 bulbs has 90 bulbs right then in a sample of 8 bulbs, the probability that at least one
bulb is defective is-
FG 9 IJ 8
FG 9 IJ 8
FG 1 IJ 8
FG 1 IJ 8
(A) 1 –
H 10 K (B)
H 10 K (C)
H 10 K (D) 1–
H 10 K
Probability [31]
Q.91 The odds in favour of escape of an enemy ship are 4 : 1 . The probability that at least one ship out of
three ships gets destroyed is-
(A) 1/125 (B) 16/125 (C) 61/125 (D) 64/125

P(X  r)
Q.92 If X is binomial variate with parameters n and p, where 0 < p <1 such that P ( X  n  r )
is independent of n and r, then p equals-
(A) 1/2 (B) 1/3 (C) 1/4 (D) none of these

Q.93 Let X denote the number of times heads occur in n tosses of a f air coin.
If P (X = 4), P (X = 5) and P (X = 6) are in AP; the value of n is-
(A) 7 (B) 10 (C) 12 (D) 8

Q.94 If X f ollows a binomial distribution with parameters n = 8 and p = 1/ 2,


then P (| X – 4 | < 2) equals-

118 119 117


(A) (B) (C) (D) none
128 128 128

Q.95 A random variable has the following probability distribution-

X : 0 1 2 3 4 5 6 7
P(X): 0 2p 2p 3p p2 2p2 7p2 2p

The value of p is
(A) 1/10 (B) –1 (C) –1/10 (D) none of these

Q.96 A random variable X has the distribution-


X 2 3 4
P(X = x) 0.3 0.4 0.3
Then, variance of the distribution is
(A) 0.6 (B) 0.7 (C) 0.77 (D) 1.55
Q.97 A fair die is thrown twenty times. The probability that on the tenth throw the fourth six appears is-
20
C10  5 6 120  5 7 84  5 6
(A) (B) (C) (D) none of these
6 20 610 610
Q.98 A bag contains 4 tickets numbered 1, 2, 3, 4 and another bag contains 6 tickets numbered 2, 4, 6,
7, 8, 9. One bag is chosen and a ticket is drawn. The probability that the ticket bears the number
4 is-
(A) 1/48 (B) 1/8 (C) 5/24 (D) None of these

Q.99 The chance of India winning toss is 3/4. If it wins the toss, then its chance of victory is 4/5 otherwise
it is only 1/2. Then chance of India’s victory is-
(A) 1/5 (B) 3/5 (C) 3/40 (D) 29/40

Q.100 Three groups A, B, C are competing for positions of dthe Board of Directors of a company. The
probabilities of their winning are 0.5, 0.3, 0.2 respectively. If the group A wins, the probability of
introducing a new product is 0.7 and the corresponding probabilities for group B and C are 0.6 and
0.5 respectively. The probability that the new product will be introduced, is-
(A) 0.18 (B) 0.35 (C) 0.10 (D) 0.63

Q.101 A bag A contains 2 white and 3 red balls and bag B contains 4 white and 5 red balls. One ball is drawn
at random from a randomly chosen bag and is found to be red. The probability that it was drawn from
bag B was-
(A) 5/14 (B) 5/16 (C) 5/18 (D) 25/52
Probability [32]
Q.102 A man is known to speak the truth 3 out of 4 times. He throws a die and reports that it is a six. The
probability that it is actually a six, is-
(A) 3/8 (B) 1/5 (C) 3/4 (D) None of these

1, 1, 1
Q.103 The probability of defective screws in three boxes A,B,C are respectively. A box is selected at
5 6 7
random and a screw drawn from it at random is found to be defective. Then the probability that it came from
box A is-
(A) 16/29 (B) 1/15 (C) 27/59 (D) 42/107

Q.104 Three letters are written to three different persons and their addresses are written of three envelopes.
The probability that letters are placed in right envelopes without seeing the addresses is -
(A) 1/27 (B) 1/6 (C) 1/9 (D) None of these
Q.105 3 letters are placed in 3 envelopes randomly. The probability that all letters are not in right envelopes
is-
(A) 1/6 (B) 1/2 (C) 1/3 (D) 5/6

Q.106 There are n letters and n addressed envelopes. The probability that all the letters are not kept in the
right envelope, is-
1 1 1
(A) (B) 1– (C) 1 – (D) None of these
n! n! n

Probability [33]
LEVEL # 2
Q.1 The probability of getting a number chosen from 1, 2,.....100 as cube is-
(A) 1/25 (B)2/25 (C) 3/25 (D) 4/25

Q.2 The probability that a random arrangement of letters i,i,i,t,t,n,n,o,a,v in row result in a word “Invitation”
is-
(A) 1/151200 (B) 1/1128800 (C) 1/24 (D) None of these

Q.3 From a group of 5 boys and 3 girls three persons are chosen at random. The probability that there are
more girls than boys is-
(A) 4/7 (B) 3/8 (C) 2/7 (D) 5/8

Q.4 A fair coin is tossed n times. If the probability that head occurs 6 times is equal to the probability that
head occurs 8 times, then n is equal to -
(A) 15 (B) 14 (C) 12 (D) 7

Q.5 5 persons A, B, C, D and E are in queue of a shop. The probability that A and E always together,
is-
(A) 1/4 (B) 2/3 (C) 2/5 (D) 3/5

Q.6 Three dice are thrown simultaneously. What is the probability of obtaining a total of 17 or 18-
(A) 1/9 (B) 1/72 (C) 1/54 (D) None of these

Q.7 If an integer is chosen at random from first 100 positive integers, then the probability that the chosen
number is a multiple of 4 or 6, is-
(A) 41/100 (B) 33/100 (C) 1/10 (D) None of these

Q.8 Two dice are thrown together. The probability of showing odd number on any one and multiple of 3 on
the other is-
(A) 13/36 (B) 11/36 (C) 1/4 (D) 2/3

Q.9 A pack of cards contains 4 aces, 4 kings, 4 queens and 4 jacks. Two cards are drawn at random.
The probability that at least one of them is an ace is-
(A) 1/5 (B) 3/16 (C) 9/20 (D) 1/9
Q.10 A committee consists of 9 experts taken from three colleges, A, B and C; of which 2 are from A,3
from B and 4 from C. If three experts resign, then the probability that they belong to different institutions
is-
(A) 1/729 (B) 2/7 (C) 1/21 (D) 1/24

Q.11 A fair coin is tossed a fixed number of times. If the probability of getting 7 heads is equal to that of
getting 9 heads, then the probability of getting 3 heads is -
35 35 7
(A) 12 (B) 14 (C) (D) None of these
2 2 212

Q.12 There are 6 positive and 8 negative numbers. Four numbers are chosen at random and multiplied. The
probability that a product is a positive number is-
(A) 505/1001 (B) 420/1001 (C) 15/1001 (D) 70/1001

Q.13 If all letters of the word ‘MISSISSIPPI’ are arranged then the probability that all S come together will be-
(A) 1/165 (B) 4/165 (C) 8/165 (D) None of these

Probability [34]
Q.14 From the word ‘POSSESSIVE’, a letter is chosen at random. The probability of it to be S is-
(A) 3/10 (B) 4/10 (C) 3/6 (D) 4/6

Q.15 Three letters are selected at random from the word ‘SLEEPER’. The probability that at least two E’s
occur is-
(A) 12/35 (B) 13/35 (C) 14/35 (D)15/35

Q.16 The probability of occurring of two events A and B are 0.21 and 0.49 respectively and of occurring both
simultaneously is 0.16, then the probability that none of the two occur is-
(A) 0.30
(B) 0.46
(C) 0.14
(D) None of these
Q.17 If A1,A2,......,An are any n events, then-
(A) P (A1  A2  ....  An) = P(A1) + P(A2) +...+ P (An)
(B) P (A1  A2  ....  An) > P(A1) + P(A2) +...+ P (An)
(C) P (A1  A2  ....  An)  P(A1) + P(A2) +...+ P (An)
(D) None of these

Q.18 Two dice are thrown, the probability of getting sum as neither 7 nor 11, is-
(A) 8/9 (B) 2/9
(C) 7/9 (D) 1/18

Q.19 Three athlete A,B and C participate in a race competition. The probability of winning A and winning
of B is twice of winning C. Then the probability that the race win by A or B, is-
(A) 2/3 (B) 1/2
(C) 4/5 (D) 1/3

Q.20 Two dice are thrown. What is the probability that the sum of the numbers appearing on the two dice
is 11, if 5 appears on the first-
(A) 1/36 (B) 1/6
(C) 5/6 (D) None of these

Q.21 A and B throw two dice, if A throws 8, the probability that B will throw a higher number is-
(A) 5/18 (B) 3/18
(C) 7/18 (D) 1/18

Q.22 Two integers are selected from integers 1 to 11. If their sum is even then the probability that both are
odd will be-
(A) 2/5 (B) 3/5 (C) 4/5 (D) None of these

Q.23 A and B toss a coin alternatively on the understanding that the first who obtains tail wins. If A starts,
what is his chance of winning ?
(A) 2/3 (B) 1/3 (C) 1/4 (D) 3/4

Q.24 A man alternately tosses a coin and throws a dice beginning with the coin. The probability that he gets
a head in the coin before he gets a 5 or 6 in the dice is -
(A) 3/4 (B) 1/2 (C) 1/3 (D) None of these
Q.25 Two dice are thrown thrice. The probability that first throw shows 10, second 11 and third 12 will be-
(A) 1/216 (B) 1/432 (C) 1/7776 (D) 1/648

Probability [35]
Q.26 A draws two cards with replacement from a well shuffled pack of cards and at the same time B throws
a pair of dice. The probability that A gets two kings and B gets a doublet is-
(A) 1/69 (B) 175/1014
(C) 1/1014 (D) None of these

Q.27 Two coins and a die are tossed. The probability that both coins fall heads and the die shows a 3 or
6 is-
(A) 1/8 (B) 1/12
(C) 1/16 (D) None of these

Q.28 A bag contains 3 white and 3 black balls. Balls are drawn one by one with out replacing them in the
bag. The probability that drawing ball will be in alternate colours is-
(A) 1/10 (B) 5/21
(C) 1/2 (D) None of these

Q.29 A bag contains 5 black and 3 blue balls. Balls are drawn (without replacement) one by one. The
probability of getting blue ball first time in fifth draw is-
(A) 1/56 (B) 3/56 (C) 4/56 (D) 5/56

Q.30 A box contains 2 black, 4 white and 3 red balls. One ball is drawn at random from the box and kept
aside. From the remaining balls in the box, another ball is drawn at random and kept aside the first.
This process is repeated till all the balls are drawn from the box. The probability that the balls drawn are
in the sequence of 2 black, 4 white and 3 red is-
(A) 1/1260 (B) 1/7560 (C) 1/126 (D) None of these

Q.31 Cards are drawn one after another till an ace is obtained. The probability of not getting ace in first 26
cards is-
(A) 46/153
(B) 109/53
(C) 23/27
(D) None of these

Q.32 ‘A’ draws two cards with replacement from a pack of 52 cards and ‘B’ throws a pair of dice what is
the chance that ‘A’ gets both cards of same suit and ‘B’ gets total of 6-
(A) 1/144 (B) 1/4
(C) 5/144 (D) 7/144

Q.33 The chance of winning a test match by India against Westindies is 1/2. A series of 5 test matches
is played and the results of all test matches are independent, then the probability of second win by
India is third test match is (No match ends in draw) -
(A) 2/3 (B) 1/2
(C) 1/4 (D) 1/8

Q.34 Odds 8 to 5 against a person who is 40 years old living till he is 70 and 4 to 3 against another person
now 50 till he will be living 80. Probability that one of them will be alive next 30 years-
(A) 59/91 (B) 44/91 (C) 51/91 (D) 32/91

Q.35 Let E and F be two independent events. The probability that both E and F happens, is
1/12 and the probability that neither E nor F happens is 1/2, then-
(A) P(E) = 1/3, P(F) = 1/4
(B) P(E) = 1/3, P(F) = 1/6
(C) P(E) = 1/6, P(F) = 1/2
(D) P(E) = 1/4, P(F) = 1/5

Probability [36]
Q.36 A digit is chosen out of digits 1, 2, 3, 4 and 5. A second digit is chosen from the remaining four digits,
then the probability that an odd digit will be selected both the times is-
(A) 1/10 (B) 3/10
(C) 3/200 (D) None of these
Q.37 The items produced by a firm are supposed to contain 5% defective items. The probability that a
sample of 8 items will contain less than 2 defective items, is-
FG IJ
27 19
7
533 19 FG IJ 6
153 1FG IJ 7
FG IJ
35 1
6
(A)
H K
20 20
(B)
400 20 H K (C)
20 20H K (D)
H K
16 20

Q.38 Cards are drawn one-by-one at random from a well shuffled pack of 52 playing cards until 2 aces are obtained
for the first time. The probability that 18 draws are required for this, is-
(A) 3/34 (B) 17/455
(C) 561/15925 (D) None of these

Q.39 A letter is taken out at random from ‘ASSISTANT’ and another is taken out from ‘STATISTICS’. The
probability that they are the same letters is-
(A) 1/45 (B) 13/90
(C) 19/90 (D) None of these

Q.40 A six faced die is a biased one. It is thrice more likely to show an odd number than show an even
number. It is thrown twice. The probability that the sum of the numbers in the two throws is even, is-
(A) 5/9 (B) 5/8 (C) 1/2 (D) None of these

Q.41 Two dice are tossed 6 times. Then the probability that sum 7 will show an exactly four of the tosses
is-
(A) 225/18442 (B) 116/20003 (C) 125/15552 (D) None of these

Q.42 The probability that a bulb produced by a factory will fuse after 150 days of use is 0.05. What is the
probability that out of 5 such bulbs none will fuse after 150 days of use-
F 19 I
(A) 1– G J
5
FG 19 IJ 5
F 3I
(C) G J
5
F 1I
(D) 90 G J
5

H 20 K (B)
H 20 K H 4K H 4K
Q.43 In order to get at least once a head with probability  0.9, the minimum number of times a coin need to
be tossed is-
(A) 3 (B) 4 (C) 5 (D) None of these

Probability [37]
LEVEL # 3
Q.1 The probability that the birth days of six persons will fall in exactly two calendar months is-

12 26 12 26  1 341
(A) 1/6 (B) C2  (C) C2  (D)
12 6
12 6 125

1 4p 1 p 1  2p
Q.2 If , , are probabilities of three mutually exclusive events, then-
4 4 2
1 1 1 2 1 1
(A) p (B) p (C) p (D) none of these
3 2 2 3 6 2

Q.3 A bag contains 50 tickets numbered 1, 2, 3, ..., 50 of which five are drawn at random and arranged in
ascending order of magnitude (x1 < x2 < x3 < x4 < x5). The probability that x3 = 30 is-
20 29 20
C2 C2 C 2 29 C2
(A) 50 (B) 50 (C) 50
(D) none of these
C5 C5 C5

Q.4 A man takes a step forward with probability 0.4 and backward with probability 0.6.The probability that at
the end of eleven steps he is one step away from the starting point is -
11
(A) C6 (0.24 )5 (B) 11
C6 (0.4) 6 0.6 5 (C) 11
C6 (0.6) 6 0.4 5 (D) none of these.

Q.5 A biased die is tossed and the respective probabilities for various faces to turn up are-

Face : 1 2 3 4 5 6
Probability: 0.1 0.24 0.19 0.18 0.15 0.14

If an even face has turned up, then the probability that it is face 2 or face 4, is
(A) 0.25 (B) 0.42 (C) 0.75 (D) 0.9

Q.6 For three events A, B and C, P (exactly one of the events A or B occurs) = P (exactly one of the
events B or C occurs) = P (exactly one of the events C or A occurs)= p and P (all the three events
occur simultaneously) = p2 ,where 0 < p < 1/2. Then the probability of atleast one of the three events
A,B and C occurring is-
3p  2p 2 p  3p 2 3p  p 2 3p  2p 2
(A) (B) (C) (D)
2 2 2 4
B
 
Q.7 If A and B are two events such that P (A) = 1/3, P(B) = 1/4 and P (A  B)= 1/5, then P   
A
(A) 37/40 (B) 37/45 (C) 23/40 (D) None

Q.8 There are 9999 tickets bearing numbers 0001, 0002,...., 9999. If one ticket is selected from these
tickets at random, the probability that the number on the ticket will consist of all different digits, is-
(A) 5040 / 9999 (B) 5000 / 9999 (C) 5030 / 9999 (D) none of these

Q.9 All possible 6 letter words each containing all the letters of the word EMHORT are formed and are
placed in a dictionary order. 10 consecutive words are now drawn at random. The probability that the
word 'MOTHER' will be among the drawn words, is
(A) 1/72 (B) 10/711 (C) 10/719 (D) none

Q.10 A number is chosen at random from the numbers 10 to 99. By seeing the number a man will laugh if
product of digits is 12. If he chooses three numbers with replacement then the probability that he will
laugh atleast once is-
3 3 3 3
3  43   4   43 
(A) 1    (B)   (C) 1    (D) 1   
5  45   25   45 
Probability [38]
(Q.11 to Q.13)
Each of the questions given below consist of Statement – I and Statement – II. Use the following Key to
choose the appropriate answer.
(A) If both Statement- I and Statement- II are true, and Statement - II is the correct explanation of
Statement– I.
(B) If both Statement - I and Statement - II are true but Statement - II is not the correct explanation of
Statement – I.
(C) If Statement - I is true but Statement - II is false.
(D) If Statement - I is false but Statement - II is true.
i
Q.11 Statement-I : Given Ei , i = 1,2,.........n are n independent events, such that P ( Ei ) = , 1  i  n, then
1 i
n
the probability that none of the n events occur is .
n1
Statement-II : Probability of occurence of all independent events together is equal to the product of the
probabilities of these events.
Q.12 Statement-I : The probability of being at least one white ball selected from two balls drawn simultaneously
34
from the bag containing 7 black and 4 white balls is .
55
Statement-II : Sample space = 11C2 = 55 No. of favourable cases = 4C1 × 7C1 + 4C2 × 7C0 = 34
34
required probability = .
55

Q.13 Statement-I : A lottery agent sells 100 tickets and award 10 prizes. If Gopal buys 10 tickets, the probability

210  1
of his winning a prize is 100
C10
Statement-II : If P(A) = 0.8, P(B) = 0.5, the greatest value of P(AB) is 0.5.

Probability [39]
EXERCISE-I
Q.1 Let a die be weighted so that the probability of a number appearing when the die is tossed is proportional
to that number. Find the probability that,
(i) An even or a prime number appears (ii) An odd prime number appears
(iii) An even composite number appears (iv) An odd composite number appears.

Q.2 Numbers are selected at random , one at a time, from the two digit numbers 00, 01, 02, ..... , 99 with
replacement. An event E occurs if & only if the product of the two digits of a selected number is 18.
If four numbers are selected, find the probability that the event E occurs at least 3 times.

Q.3 In a box , there are 8 alphabets cards with the letters : S, S, A, A,A, H, H, H . Find the probability
that the word ‘ASH’ will form if :
(i) the three cards are drawn one by one & placed on the table in the same order that they are drawn.
(ii) the three cards are drawn simultaneously .

Q.4 There are 2 groups of subjects one of which consists of 5 science subjects & 3 engg. subjects & other
consists of 3 science & 5 engg. subjects . An unbiased die is cast . If the number 3 or 5 turns up a subject
is selected at random from first group, otherwise the subject is selected from 2nd group . Find the
probability that an engg. subject is selected.

Q.5 A pair of fair dice is tossed. Find the probability that the maximum of the two numbers is greater than 4.

Q.6 In a building programme the event that all the materials will be delivered at the correct time is M, and the
event that the building programme will be completed on time is F . Given that P (M) = 0.8 and
P (M  F) = 0.65, find P (F/M) . If P (F) = 0.7, find the probability that the building programme will
be completed on time if all the materials are not delivered at the correct time .

Q.7 In a given race, the odds in favour of four horses A, B, C & D are 1 : 3, 1 : 4, 1 : 5 and 1 : 6 respectively.
Assuming that a dead heat is impossible, find the chance that one of them wins the race.

Q.8 A covered basket of flowers has some lilies and roses. In search of rose, Sweety and Shweta alternately
pick up a flower from the basket but puts it back if it is not a rose. Sweety is 3 times more likelyto be the
first one to pick a rose. If sweety begin this 'rose hunt' and if there are 60 lilies in the basket, find the
number of roses in the basket.

Q.9 Least number of times must a fair die be tossed in order to have a probability of at least 91/216,
of getting atleast one six.

Q.10 Suppose the probability for A to win a game against B is 0.4. If A has an option of playing either a
“BEST OF THREE GAMES” or a “BEST OF 5 GAMES” match against B , which option should A choose
so that the probability of his winning the match is higher? (No game ends in a draw).

Q.11 A room has three electric lamps . From a collection of 10 electric bulbs of which 6 are good 3 are
selected at random & put in the lamps. Find the probability that the room is lighted.

Q.12 A bomber wants to destroy a bridge . Two bombs are sufficient to destroy it.
If four bombs are dropped, what is the probability that it is destroyed, if the chance of a bomb hitting the
target is 0.4.

Probability [40]
Q.13 The chance of one event happening is the square of the chance of a 2nd event, but odds against the first
are the cubes of the odds against the 2nd . Find the chances of each. (assume that both events are neither
sure nor impossible).

Q.14 A box contains 5 radio tubes of which 2 are defective . The tubes are tested one after the other until the
2 defective tubes are discovered . Find the probability that the process stopped on the
(i) Second test; (ii) Third test. If the process stopped on the third test , find the probability that the first
tube is non defective.

Q.15 Anand plays with Karpov 3 games of chess. The probability that he wins a game is 0.5, looses with
probability 0.3 and ties with probability 0.2. If he plays 3 games then find the probability that he wins
atleast two games.

Q.16 An aircraft gun can take a maximum of four shots at an enemy’s plane moving away from it. The probability
of hitting the plane at first, second, third & fourth shots are 0.4, 0.3, 0.2 & 0.1 respectively. What is the
probability that the gun hits the plane .

Q.17 In a batch of 10 articles, 4 articles are defective. 6 articles are taken from the batch for inspection.
If more than 2 articles in this batch are defective , the whole batch is rejected Find the probability that
the batch will be rejected.

Q.18 Given P(AB) = 5/6 ; P(AB) = 1/3 ; P( B ) = 1/2. Determine P(A) & P(B). Hence show that the
events A & B are independent.

Q.19 One hundred management students who read at least one of the three business magazines are surveyed
to study the readership pattern. It is found that 80 read Business India, 50 read
Business world and 30 read Business Today. Five students read all the three magazines. A student was
selected randomly. Find the probability that he reads exactly two magazines.

Q.20 An author writes a good book with a probability of 1/2. If it is good it is published with a probability of
2/3. If it is not, it is published with a probability of 1/4. Find the probability that he will get atleast one
book published if he writes two.

Q.21 3 students {A, B, C} tackle a puzzle together and offers a solution upon which majority of the 3 agrees.
Probability ofA solving the puzzle correctly is p. Probability of B solving the puzzle correctly is also p. C
is a dumb student who randomly supports the solution of either A or B. There is one more student D,
whose probability of solving the puzzle correctly is once again, p. Out of the 3 member team {A, B, C}
and one member team {D}, Which one is more likely to solve the puzzle correctly.

Q.22 A uniform unbised die is constructed in the shape of a regular tetrahedron with faces numbered 2, 2, 3
and 4 and the score is taken from the face on which the die lands. If two such dice are thrown together,
find the probability of scoring.
(i) exactly 6 on each of 3 successive throws.
(ii) more than 4 on at least one of the three successive throws.

Q.23 Two cards are drawn from a well shuffled pack of 52 cards. Find the probability that one of them
is a red card & the other is a queen.

Q.24 A cube with all six faces coloured is cut into 64 cubical blocks of the same size which are thoroughly
mixed. Find the probability that the 2 randomly chosen blocks have 2 coloured faces each.
Probability [41]
Q.25 Consider the following events for a family with children
A = {of both sexes} ; B = {at most one boy}
In which of the following (are/is) the events A and B are independent.
(a) if a family has 3 children (b) if a family has 2 children
Assume that the birth of a boy or a girl is equally likely mutually exclusive and exhaustive.

Q.26 A player tosses an unbiased coin and is to score two points for every head turned up and one point for
every tail turned up. If Pn denotes the probability that his score is exactly n points, prove that
1
Pn – Pn – 1 = (P – Pn – 1 ) n> 3
2 n–2
Also compute P1 and P2 and hence deduce the pr that he scores exactly 4.

Q.27 Each of the ‘n’ passengers sitting in a bus may get down from it at the next stop with probability
p . Moreover , at the next stop either no passenger or exactly one passenger boards the bus . The
probability of no passenger boarding the bus at the next stop being po . Find the probability that
when the bus continues on its way after the stop , there will again be ‘n’ passengers in the bus.

Q.28 The difference between the mean & variance of a Binomial Variate ‘X’ is unity & the difference of their
square is 11. Find the probability distribution of ‘X’.

Q.29 An examination consists of 8 questions in each of which the candidate must say which one of the
5 alternatives is correct one . Assuming that the student has not prepared earlier chooses for each of the
question any one of 5 answers with equal probability.
(i) prove that the probability that he gets more than one correct answer is (58  3 x 48) / 58 .
(ii) find the probability that he gets correct answers to six or more questions.
(iii) find the standard deviation of this distribution.

Q.30 Two bad eggs are accidently mixed with ten good ones. Three eggs are drawn at random without
replacement, from this lot. Compute mean & S.D. for the number of bad eggs drawn.

EXERCISE-II

Q.1 The probabilities that three men hit a target are, respectively, 0.3, 0.5 and 0.4. Each fires once at the
target. (As usual, assume that the three events that each hits the target are independent)
(a) Find the probability that they all : (i) hit the target ; (ii) miss the target
(b) Find the probability that the target is hit : (i) at least once, (ii) exactly once.
(c) If only one hits the target, what is the probability that it was the first man?

Q.2 Let A & B be two events defined on a sample space . Given P(A) = 0.4 ; P(B) = 0.80 and
P  A / B = 0.10. Then find ; (i) P  A  B & 
P  A  B   A  B  . 
Q.3 Three shots are fired independently at a target in succession. The probabilities that the target is hit in the
first shot is 1/2 , in the second 2/3 and in the third shot is 3/4. In case of exactly one hit , the probability
of destroying the target is 1/3 and in the case of exactly two hits, 7/11 and in the case of three hits is
1.0. Find the probability of destroying the target in three shots.
Q.4 In a game of chance each player throws two unbiased dice and scores the difference between the larger
and smaller number which arise . Two players compete and one or the other wins if and only if he scores
atleast 4 more than his opponent . Find the probability that neither player wins .

Probability [42]
Q.5 A certain drug , manufactured by a Company is tested chemically for its toxic nature. Let the event
"THE DRUG IS TOXIC" be denoted by H & the event "THE CHEMICAL TEST REVEALS THAT THE DRUG
IS TOXIC" be denoted by S. Let P(H) = a, P  S / H = P  S / H  = 1  a. Then show that the probability
that the drug is not toxic given that the chemical test reveals that it is toxic, is free from ‘a’.

Q.6 A plane is landing. If the weather is favourable, the pilot landing the plane can see the runway. In this case
the probability of a safe landing is p1. If there is a low cloud ceiling, the pilot has to make a blind landing by
instruments. The reliability (the probability of failure free functioning) of the instruments needed for a blind
landing is P. If the blind landing instruments function normally, the plane makes a safe landing with the same
probability p1 as in the case of a visual landing. If the blind landing instruments fail, then the pilot may make
a safe landing with probability p2 < p1. Compute the probability of a safe landing if it is known that in K
percent of the cases there is a low cloud ceiling. Also find the probability that the pilot used the blind landing
instrument, if the plane landed safely.

Q.7 A train consists of n carriages , each of which may have a defect with probability p. All the carriages
are inspected , independently of one another , by two inspectors ; the first detects defects (if any) with
probability p1 , & the second with probability p2 . If none of the carriages is found to have a defect,
the train departs . Find the probability of the event ; " THE TRAIN DEPARTS WITH ATLEAST ONE
DEFECTIVE CARRIAGE ".

Q.8 A is a set containing n distinct elements. A non-zero subset P of A is chosen at random. The set A is
reconstructed by replacing the elements of P. A non-zero subset Q of A is again chosen at random.
Find the probability that P & Q have no common elements.

Q.9 In a multiple choice question there are five alternative answers of which one or more than one is correct. A
candidate will get marks on the question only if he ticks the correct answers. The candidate ticks the
answers at random. If the probability of the candidate getting marks on the question is to be greater than or
equal to 1/3 find the least number of chances he should be allowed.

Q.10 n people are asked a question successively in a random order & exactly 2 of the n people know
the answer :
(a) If n > 5, find the probability that the first four of those asked do not know the answer.
(b) Show that the probability that the rth person asked is the first person to know the answer is :
 2 (n  r) 
  , if 1 < r < n .
 n ( n  1) 

Q.11 A box contains three coins two of them are fair and one two  headed. A coin is selected at random and
tossed. If the head appears the coin is tossed again, if a tail appears, then another coin is selected from
the remaining coins and tossed.
(i) Find the probability that head appears twice.
(ii) If the same coin is tossed twice, find the probability that it is two headed coin.
(iii) Find the probability that tail appears twice.

Q.12 The ratio of the number of trucks along a highway, on which a petrol pump is located, to the number of
cars running along the same highway is 3 : 2. It is known that an average of one truck in thirty trucks and
two cars in fifty cars stop at the petrol pump to be filled up with the fuel. If a vehicle stops at the petrol
pump to be filled up with the fuel, find the probability that it is a car.

Probability [43]
Q.13 A batch of fifty radio sets was purchased from three different companies A, B and C. Eighteen of them
were manufactured by A, twenty of them by B and the rest were manufactured by C.
The companies A and C produce excellent quality radio sets with probability equal to 0.9 ; B produces
the same with the probability equal to 0.6.
What is the probability of the event that the excellent quality radio set chosen at random is manufactured
by the company B?

Q.14 The contents of three urns 1, 2 & 3 are as follows :


1 W, 2 R, 3B balls
2 W, 3 R, 1B balls
3 W, 1 R, 2B balls
An urn is chosen at random & from it two balls are drawn at random & are found to be
"1 RED & 1 WHITE ". Find the probability that they came from the 2nd urn.

m
Q.15 Suppose that there are 5 red points and 4 blue points on a circle. Let be the probability that a convex
n
polygon whose vertices are among the 9 points has at least one blue vertex where m and n are relatively
prime. Find (m + n).

Q.16 There are 6 red balls & 8 green balls in a bag . 5 balls are drawn out at random & placed in a red box ; the
remaining 9 balls are put in a green box . What is the probability that the number of red balls in the green box
plus the number of green balls in the red box is not a prime number?

Q.17 Two cards are randomly drawn from a well shuffled pack of 52 playing cards, without replacement. Let
x be the first number and y be the second number.
Suppose that Ace is denoted by the number 1; Jack is denoted by the number 11 ; Queen is denoted
by the number 12 ; King is denoted by the number 13.
Find the probability that x and y satisfy log3(x + y) – log3x – log3y + 1 = 0.

Q.18(a) Two numbers x & y are chosen at random from the set {1,2,3,4,....3n}. Find the probability that
x²  y² is divisible by 3 .
(b) If two whole numbers x and y are randomly selected, then find the probability that x3 + y3 is divisible by 8.
a2
Q.19 A hunter’s chance of shooting an animal at a distance r is 2 (r > a) . He fires when r = 2a &
r
if he misses he reloads & fires when r = 3a, 4a, ..... If he misses at a distance ‘na’, the animal escapes.
Find the odds against the hunter.

Q.20 An unbiased normal coin is tossed 'n' times. Let :


E1 : event that both Heads and Tails are present in 'n' tosses.
E2 : event that the coin shows up Heads atmost once.
Find the value of 'n' for which E1 & E2 are independent.
n2
Q.21 A coin is tossed (m + n) times (m>n). Show that the probability of at least m consecutive heads is
2 m1

Q.22 There are two lots of identical articles with different amount of standard and defective articles. There are
N articles in the first lot, n of which are defective and M articles in the second lot, m of which are
defective. K articles are selected from the first lot and L articles from the second and a new lot results.
Find the probability that an article selected at random from the new lot is defective.
Probability [44]
Q.23 m red socks and n blue socks (m > n) in a cupboard are well mixed up, where m + n  101. If two socks
are taken out at random, the chance that they have the same colour is 1/2. Find the largest value of m.

Q.24 With respect to a particular question on a multiple choice test (having 4 alternatives with only 1 correct)
a student knows the answer and therefore can eliminate 3 of the 4 choices from consideration with
probability 2/3, can eliminate 2 of the 4 choices from consideration with probability 1/6, can eliminate 1
choice from consideration with probability 1/9, and can eliminate none with probability 1/18. If the
student knows the answer, he answers correctly, otherwise he guesses from among the choices not
eliminated. a
If the answer given by the student was found correct, then the probability that he knew the answer is
b
where a and b are relatively prime. Find the value of (a + b).

Q.25 In a knockout tournament 2n equally skilled players; S1, S2, ............. S2n are participating. In each
round players are divided in pairs at random & winner from each pair moves in the next round. If S2
reaches the semifinal then find the probability that S1 wins the tournament.

EXERCISE-III Section - A

Q.1 If p & q are chosen randomly from the set {1, 2, 3, 4, 5, 6, 7, 8, 9, 10} with replacement. Determine
the probability that the roots of the equation x2 + px + q = 0 are real. [ JEE '97, 5 ]

Q.2 There is 30% chance that it rains on any particular day . What is the probability that there is at least one
rainy day within a period of 7  days ? Given that there is at least one rainy day, what is the probability
that there are at least two rainy days ? [ REE '97, 6 ]

Q.3 Select the correct alternative(s) . [ JEE '98, 6 × 2 = 12 ]


(i) 7 white balls & 3 black balls are randomly placed in a row. The probability that no two black balls are
placed adjacently equals :
(A) 1/2 (B) 7/15 (C) 2/15 (D) 1/3

(ii) If from each of the 3 boxes containing 3 white & 1 black, 2 white & 2 black, 1 white & 3 black balls,
one ball is drawn at random, then the probability that 2 white & 1 black ball will be drawn is :
(A) 13/32 (B) 1/4 (C) 1/32 (D) 3/16

(iii) If E & F are the complementary events of events E & F respectively & if 0 < P (F) < 1, then :
(A) P (EF) + P( E F) = 1 (B) P (EF) + P(E F ) = 1
(C) P ( E F) + P (E F ) = 1 (D) P (E F ) + P ( E  F ) = 1

(iv) There are 4 machines & it is known that exactly 2 of them are faulty . They are tested, one by one, in a
random order till both the faulty machines are identified . Then the probability that only 2 tests are
needed is :
(A) 1/3 (B) 1/6 (C) 1/2 (D) 1/4

(v) If E & F are events with P(E)  P(F) & P(E  F) > 0, then :
(A) occurrence of E  occurrence of F
(B) occurrence of F  occurrence of E
(C) non  occurrence of E  non  occurrence of F
(D) none of the above implications holds.
Probability [45]
(vi) A fair coin is tossed repeatedly. If tail appears on first four tosses, then the probability of head
appearing on fifth toss equals :
(A) 1/2 (B) 1/32 (C) 31/32 (D) 1/5

Q.4 3 players A, B & C toss a coin cyclically in that order (that is A, B, C, A, B, C, A, B, ......) till a head
shows . Let p be the probability that the coin shows a head. Let ,  &  be respectively the
probabilities that A, B and C gets the first head . Prove that
= (1  p). Determine ,  &  (in terms of p). [ JEE '98, 8 ]

Q.5 Each coefficient in the equation ax2 + bx + c = 0 is determined by throwing an ordinary die . Find the
probability that the equation will have equal roots. [ REE '98, 6 ]

Q.6(a) If the integers m and n are chosen at random between 1 and 100, then the probability that a number of
the form 7m + 7n is divisible by 5 equals
1 1 1 1
(A) (B) (C) (D)
4 7 8 49

(b) The probability that a student passes in Mathematics, Physics and Chemistry are m, p and c respectively.
Of these subjects, the student has a 75% chance of passing in at least one, a 50% chance of passing in
at least two, and a 40% chance of passing in exactly two, which of the following relations are true?
19 27 1 1
(A) p + m + c = (B) p + m + c = (C) pmc = (D) pmc =
20 20 10 4

(c) Eight players P1, P2, P3, ............P8 play a knock-out tournament. It is known that whenever the players
Pi and Pj play, the player Pi will win if i < j. Assuming that the players are paired at random in each round,
what is the probability that the player P4 reaches the final. [ JEE ' 99, 2 + 3 + 10 (out of 200)]

Q.7 Four cards are drawn from a pack of 52 playing cards. Find the probability (correct upto two places of
decimals) of drawing exactly one pair. [REE'99, 6]

Q.8 A coin has probability ' p ' of showing head when tossed. It is tossed 'n' times. Let pn denote the
probability that no two (or more) consecutive heads occur. Prove that,
p1 = 1 , p2 = 1  p2 & pn = (1  p) pn  1 + p (1  p) pn  2 , for all n  3.
[ JEE ' 2000 (Mains), 5 ]

Q.9 A and B are two independent events. The probability that both occur simultaneously is 1/6 and the
probability that neither occurs is 1/3. Find the probabilities of occurance of the events A and B separately.
[ REE ' 2000 (Mains), 3]

Q.10 Two cards are drawn at random from a pack of playing cards. Find the probability that one card is a
heart and the other is an ace. [ REE ' 2001 (Mains), 3 ]

Q.11(a) An urn contains 'm' white and 'n' black balls. A ball is drawn at random and is put back into the urn along
with K additional balls of the same colour as that of the ball drawn. A ball is again drawn at random.
What is the probability that the ball drawn now is white.

(b) An unbiased die, with faces numbered 1, 2, 3, 4, 5, 6 is thrown n times and the list of n numbers showing
up is noted. What is the probability that among the numbers 1, 2, 3, 4, 5, 6, only three numbers appear
in the list. [JEE ' 2001 (Mains), 5 + 5 ]

Probability [46]
Q.12 A box contains N coins, m of which are fair and the rest are biased. The probability of getting a head
when a fair coin is tossed is 1/2, while it is 2/3 when a biased coin is tossed. A coin is drawn from the box
at random and is tossed twice. The first time it shows head and the second time it shows tail. What is the
probability that the coin drawn is fair? [ JEE ' 2002 (mains)]

Q.13(a) A person takes three tests in succession. The probability of his passing the first test is p, that of his
passing each successive test is p or p/2 according as he passes or fails in the preceding one. He gets
selected provided he passes at least two tests. Determine the probability that the person is selected.

(b) In a combat, A targets B, and both B and C target A. The probabilities of A, B, C hitting their targets
are 2/3 , 1/2 and 1/3 respectively. They shoot simultaneously and A is hit. Find the probability that B hits
his target whereas C does not. [JEE' 2003, Mains-2 + 2 out of 60]

Q.14(a) Three distinct numbers are selected from first 100 natural numbers. The probability that all the three
numbers are divisible by 2 and 3 is
4 4 4 4
(A) (B) (C) (D)
25 35 55 1155

(b) If A and B are independent events, prove that P (A  B) · P (A'  B')  P (C), where C is an event
defined that exactly one of A or B occurs.

(c) A bag contains 12 red balls and 6 white balls. Six balls are drawn one by one without replacement of
which atleast 4 balls are white. Find the probability that in the next two draws exactly one white ball is
drawn (leave the answer in terms of nCr). [JEE 2004, 3 + 2 + 4]

Q.15(a) A six faced fair dice is thrown until 1 comes, then the probability that 1 comes in even number of trials
is
(A) 5/11 (B) 5/6 (C) 6/11 (D) 1/6
[JEE 2005 (Scr)]
1 3 2 1
(b) A person goes to office either by car, scooter, bus or train probability of which being , , and
7 7 7 7
2 1 4 1
respectively. Probability that he reaches office late, if he takes car, scooter, bus or train is , , and
9 9 9 9
respectively. Given that he reached office in time, then what is the probability that he travelled by a car.
[JEE 2005 (Mains), 2]

Comprehension (3 questions)
There are n urns each containing n + 1 balls such that the ith urn contains i white balls and (n + 1 – i) red
balls. Let ui be the event of selecting ith urn, i = 1, 2, 3, ......, n and w denotes the event of getting a white
ball.
Q.16(a) If P(ui)  i where i = 1, 2, 3,....., n then Lim P( w ) is equal to
n 
(A) 1 (B) 2/3 (C) 3/4 (D) 1/4

(b) If P(ui) = c, where c is a constant then P(un/w) is equal to


2 1 n 1
(A) (B) (C) (D)
n 1 n 1 n 1 2

Probability [47]
1
(c) If n is even and E denotes the event of choosing even numbered urn ( P(u i )  ), then the value of Pw E  , is
n
n2 n2 n 1
(A) (B) (C) (B)
2n  1 2n  1 n 1 n 1
[JEE 2006, 5 marks each]
EXERCISE-III Section - B
MCQ's with ONE correct answer :

Probability [48]
MCQ's with ONE or MORE THAN ONE correct :

Probability [49]
Comprehension Based Questions :

Probability [50]
Probability [51]
Probability [52]
ANSWER - KEY
LEVEL # 1
Q.No. 1 2 3 4 5 6 7 8 9 10 11 12 13 14 15 16 17 18 19 20
Ans. D B B B B B B B B A C A B A D B D A B B
Q.No. 21 22 23 24 25 26 27 28 29 30 31 32 33 34 35 36 37 38 39 40
Ans. B C B B C A C B C A A C B B D C A D C C
Q.No. 41 42 43 44 45 46 47 48 49 50 51 52 53 54 55 56 57 58 59 60
Ans. B B A D D B C B A D A A D A C A D D D B
Q.No. 61 62 63 64 65 66 67 68 69 70 71 72 73 74 75 76 77 78 79 80
Ans. A A C C C B B A B D C B A A A B A B D B
Q.No. 81 82 83 84 85 86 87 88 89 90 91 92 93 94 95 96 97 98 99 100
Ans. B C B A B C B B B A C A A B A A C C D D
Q.No. 101 102 103 104 105 106
Ans. D A D B D B

LEVEL # 2
Q.No. 1 2 3 4 5 6 7 8 9 10 11 12 13 14 15 16 17 18 19 20
Ans. A A C B C C B B C B A A B B B B C C C B
Q.No. 21 22 23 24 25 26 27 28 29 30 31 32 33 34 35 36 37 38 39 40
Ans. A B A A C C B A B A D C C B A B A C C B
Q.No. 41 42 43
Ans. C B B

LEVEL # 3
Q.No. 1 2 3 4 5 6 7 8 9 10 11 12 13
A ns. D D C A C A A A B D D A B

EXERCISE-I
20 8 10
Q 1. (i) (ii) (iii) (iv) 0 Q 2. 97/(25)4 Q 3. (i) 3/56 (ii) 9/28
21 21 21
13 1
Q 4. 13/24 Q 5. 5/9 Q 6. P (F/M) = ; P (F/ M ) =
16 4
Q 7. 319/420 Q 8. 120 Q 9. 3
29 328 1 1
Q 10. best of 3 games Q 11. Q 12. Q 13. ,
30 625 9 3
Q 14. (i) 1/10, (ii) 3/10, (iii) 2/3 Q 15. 1/2 Q 16. 0.6976

Q 17. 19/42 Q 18. P(A) = 2/3, P(B) = 1/2 Q 19. 1/2

Probability [53]
125 63
Q 20. 407/576 Q 21. Both are equally likely Q 22. (i) 3 ; (ii)
16 64
24
C2 23
Q 23. 101/1326 Q 24. 64
or
C2 168

Q25. Independent in (a) and not independent in (b) Q 26. P1 = 1/2 , P2 = 3/4
36
 5 1
Q 27. (1  p)n1 . [ po (1  p) + np(1 p0 )] Q 28.  6  6 

481 4 2
Q 29. , Q 30. mean = 0.5
58 5

EXERCISE-II
5
Q 1. (a) 6%, 21% ; (b) 79%, 44%, (c) 9/44  20.45% Q 2. (i) 0.82, (ii) 0.76 Q 3.
8
Q 4. 74/81 Q 5. P  H / S = 1/2
K
[ P p1  (1  P) p 2 ]
K K 100
Q6. P( E)  (1  ) p1  [P p1  (1  P) p 2 ] ; P(H2/A) =
100 100  K  K
1   p1  [ P p1  (1  P) p 2 ]
 100  100

Q 7. 1  [ 1p (1p1) (1p2)]n Q 8. (3n-2n+1+1)/(4n-2n+1+1) Q 9. 11

(n  4) (n  5) 4
Q 10. (a) n ((n  1)
Q 11. 1/2, 1/2, 1/12 Q 12.
9
4
Q.13 Q 14. 6/11 Q 15. 458
13

11 (5n  3) 5
Q 16. 213/1001 Q.17 Q 18. (a) (9n  3) (b)
663 16

KnMLmN
Q 19. n+1 : n1 Q 20. n = 3 Q 22. M N ( K  L)

3
Q.23 55 Q.24 317 Q.25 4 2  1
n

EXERCISE-III (Section - A)

Q.1 31/50 Q.2 [1  (7/10)7  7C1 (3/10) (7/10)6] / 1  (7/10)7

Q.3 (i) B (ii) A (iii) A, D (iv) A (v) D (vi) A

Probability [54]
p (1  p)p (1  p)2 p
Q.4   ,   ,   Q.5 5/216
1  (1  p)3 1  (1  p)3 1  (1  p)3

1 1 1 1
Q.6 (a) A (b) B, C (c) 4/35 Q.7 0.31 Q.9 & or &
2 3 3 2

1 m
6
C 3  3 n  3.2 n  3 9m
Q.10 Q.11 (a) ; (b) Q.12
26 m n 6n m  8N

12
C 2 6C 410C12C1 12 C16C 511C11C1
Q.13 (a) p2 (2 – p) ; (b) 1/2 Q.14 (a) D , (c) 12
C2 
12
C 2 6C 4 12 C16C 5 12 C 0 6C 6 
1
Q.15 (a) A, (b) Q.16 (a) B, (b) A, (c) B
7
EXERCISE-III (Section - B)

Probability [55]

You might also like